You are on page 1of 101

Obstetrics & gynecology

Past year questions


th th
5 & 6 years
2015 – 2019

Done by: Fahad almutairi


& Hadiya atallah
Topics:

- Reproductive basics ( physiology and anatomy) and changed in


pregnancy
- Ectopic pregnancy
- Miscarriage
- GTN
- RH isoimmunization
- PROM and preterm
- Prolonged pregnancy & induction of labor
- multiple pregnancy
- PET
- Infection during pregnancy
- DM
- Anemia , thromboembolism and bleeding disorders
- Other medical disorders
- Drugs in pregnancy , tocolytics , analgesia and anesthesia
- PPH
- Puerperium
- Antepartum hemorrhage and obstetrics emergency
- Fetal malpresentation
- Normal and abnormal labor
- Delivery ( vaginal , assisted & CS )
- ANC
- Fetal abnormalities, growth and monitoring
- Urinary incontinence & genital prolapse
- Fibroid
- Endometriosis, Adenomyosis & Endometrial CA
- Cervical dis & CA
- Ovarian CA
- Vaginal discharge
- Menorrhagia
- Amenorrhea
- Infertility
- Contraception
- Menopause
- Obstetric & gynecology Procedures & Pap Smear
- Others
Reproductive basics ( physiology and anatomy) and changed in
pregnancy
One of the following tests is not used to A
assess ovulation:
a. Estrogen concentration on day 2 of the
All of the follwing is true except:
cycle
Answer : Brain Ct is mandatory when LH &FSH increase.
b. Mid-luteal phase serum progesterone
concentration
c. Basal Body Temperature charting
d. Urinary LH surge testing
e. Serial U/S scan
B Regarding follicle-stimulating hormone A
(FSH), only one of the following
statements
Which of the following parameters doesn’t decrease in a
is correct:
normal pregnancy a. Is responsible for estradiol production
a. Platelet count from the granulosa cells
b. Packed cell volume b. Brings about follicular rupture
c. Gastric emptying c. Is raised in polycystic ovary syndrome
Answer : A d. Is necessary for the initial stages of
embryo development
e. Is necessary for maintenance of the
corpus luteum
The normal lining of the fallopian tube is : A All are normal physiological changes in
a. Columnar epithelium with cilia pregnancy except:
b. Squamous epithelium a. Increase HR
c. Fibrous connective tissue b. Increase GFR
d. Transitional epithelium c. Increased gastric time emptying
e. Cuboidal epithelium d. Increase WBC
Answer: They all seem correct!
During the first half of the menstrual cycle A
the endometrium thickens and
regrows/ rebuilds under the effect of:
The last feature to develop in puberty in females is
A- Estrogen
a. Breast budding
B- Progesterone
b. Pubic hair development
C- LH
c. Menarche
D- FSH
E- GnRH
Implantation in the uterus occur at
The number of chromosomes in a normal fetus is which stage of development :
A- 45 XX a. Blastocyst.
B- 45 XY b. Zygote.
C- 46 c. Morula formation .
D- 23 d. Embryonic disk.
Answer: C e. Eight-cell embryo.
Answer: A
All are bad except:
False about lactation:
A- passed 13 week ( not sure )
Answer : Caused by drop in estrogen postnatally B- absence of pregnancy symptoms
C- maternal age > 40
All of the following increase in pregnancy except: All of the following we can do to assess
ovulation except:
A- renal blood flow
B- WBCs A- BBT chart
C- cardiac output B- ovarian biopsy
Reproductive basics ( physiology and anatomy) and changed in
pregnancy
D- peripheral vascular resistance C- mid- luteal progesterone
D- serial US
E- urine LH surge test
Uterine corpus is mainly composed of :

A- Smooth muscle What is the components of fallopian tubes :


B- Endometrium
C- Fibrous tissue
- Answer : Columnar with cilia
D- Elastic tissue
E- Estrogen receptors
Answer : A or B ?
A 165 cm tall female, which of the following is the most All the following are true changes in.pregnancy
likely type of pelvis? except
A-decrease urea creatinine
a. Gynecoid
B-glycosurea could be normal
b. Android C-decrease PVR
c. Platypeloid D-Increase substances clearance
d. Anthropoid E-decrease triglyceride
The peak secretion of progesterone in 28 days cycle
occurs on :
a. 21st day of cycle.
b. 7th day of cycle.
c. 14th day of cycle.
d. 36 hours before ovulation.
e. 1st day of cycle.
Answer: A

Ectopic pregnancy
A ectopic pregnancy , all true except:

A- Remove the affected tube by salpingectomy if


All cause ectopic pregnancy except :
the other tube is healthy
B- Methotrexate can be used if bhcg <3500
A- COCP
C- Most common site in the fallopian is the
B- mini pills
infundibulum
C- IUCD
D- Bleeding precedes abdominal pain
E- laparoscopy is the choice in stable pts of
ectopic pregnancy
Regarding ectopic pregnancy, one is true: A Regarding ectopic pregnancy, one is
true:
a. Expectant management is possible in selected
A- Mini-pill contraception is a risk factor
cases b. Pelvic mass almost always present
B- A positive urine pregnancy test is
c. Commonest in the left tube than the right tube
diagnostic
d. A positive pregnancy test and absent
C- Vaginal bleeding is usually heavy
intrauterine sac is diagnostic for ectopic pregnancy
D- Amenorrhea is present in all patients
e. Recently most cases are silent
E- Most cases are ruptured ectopic
Answer: B
About ectopic pregnancy, one is true
True about ectopic: a. can present before missed menses
b. positive pregnancy test and empty uterus on
a. +the most common site is the fallopian ampulla
ultrasound are diagnostic
c. fallopian isthmus is the most common site
Ectopic pregnancy
d. most cases are older than 35 years
e. usually asymptomatic
Regarding ectopic pregnancy, only one of the Regarding ectopic pregnancy; only one of the
following is true: following statements is correct
a. The commonest presentations are acute a. Bleeding usually precedes abdominal pain
b. Previous ectopic is a recognized b. Most patients have gradually increasing
risk factor c. Missed period is always intermittent abdominal pain
present c. A ruptured corpus luteum cyst may cause a
d. Ultrasound and β-HCG are diagnostic similar clinical picture
e. A positive urine pregnancy test is found in half d. Vaginal bleeding is essential for diagnosis e.
of cases Recurrence in future pregnancy is 25%
Answer: B Answer : C
case of ectopic pregnancy with no gestational sac ,
Case of ectopic pregnancy and she was stable what
HCG 3000 What is the next step to confirm dx:
is the next step
A- laparoscopy
A- Methotrexate
B- hestyroscopy
B- Labroscopy serum BhCG
C- serum HCG
C- transvaginal U/S ( NOT sure)
D- vaginal US

Regarding malpresentation , Which of the


Regarding ectopic pregnancy , which of the following
following is Not true ?
is NOT true?
Answer : Mode of delivery in brow presentation is
Answer: all cases treated surgically
by forceps vaginal delivery
Regarding Ectopic pregnancy, one is true?

A- More common in the outer third of the One of the following is not a sign of ruptured ectopic
fallopian tube pregnancy :
B- negative CRP can rule out ectopic A-Hypotension
pregnancy B-Tachycardia
C- More common in the left than right C-Shoulder tip
fallopian rube D-Heavy Vaginal bleeding
D- Most cases present as acute ruptured
ectopic
E- Salpingo-Oophorectomy is the
recommended treatment
Not investigation in ectopic pregnancy: best for uterine cavity and tubal patency ?
A-Progestrone Answer : HSG
B-Hysteroscope
Miscarriage
All are causes of recurrent miscarriage
All are causes of abdominal pain and vaginal bleeding after 8 wk
except :
of amenorrhea, except :
A- Chromosomal abnormalities
A- Placental abruption
B- Rubella
B- Inevitable
C- Spetated uterus
C- Incomplete miscarriage
D- Cervical incompetence
D- Ectopic pregnancy
Answer B
Answer A
One of the following does not cause Regarding cervical incompetence, which is false:
recurrent miscarriage:
A- Can be diagnosed by U/S
A. Bicornuate uterus B- Can be caused by cone biopsy of the cervix
B. Cervical incompetence C- Is a cause of preterm labour
C. Subserosal fibroid D- Is best treated with cervical suture at the end of the 1st
D. Hormonal dysfunction trimester
E. Chromosomal abnormalities E- . Is a recognized cause of first trimester abortion
Answer: E or C Answer: Is a recognized cause of first trimester abortion
A 20 weeks pregnant female presents to
17. Features of threatened miscarriage include all of the
the clinic. On ultrasound there is no following except:
a. Ultrasound is essential for the diagnosis
fetal heart beat, your next step in b. About 60% of cases continue to term if fetal heart was
management is: seen by U/S
c. The uterine size is usually corresponding to date d.
A. Vaginal PGE1 The cervix is closed
B. Oxytocin e. The typical presentation is mild vaginal bleeding with little
C. D and C or no pain
Answer: 1 Answer: B

25. Regarding recurrent miscarriages,


all are true except: a. Defined as 3 or
more consecutive induced abortions b. Medical treatment of missed miscarriage is with
Occurs in about 1% of women a. Oxytocin infusion IV
c. Can be caused by thrombophilia b. Prostaglandin F2 alpha IV
d. Can be caused by c. Misoprostol vaginally
uterine anomalies e. Is d. Ergometrine
idiopathic in about Answer C
50% of cases
Answer: A
43. Features of missed abortion include 44. Primigravida with 7 weeks of amenorrhea presents with
all of the following except:
mild abdominal pain and
a. Most cases are diagnosed accidentally
spotting and a positive pregnancy test. Abdominal US shows
during routine u/s examination
no gestational sac. She is hemodynamically stable. What is
b. Ultrasound is essential for the diagnosis
the best next step of management?
c. The best method of treatment in
the 2nd trimester is medical a. Order a serum b-hCG
termination
b. Transvaginal US
d. The uterus may be small for date
c.Laparoscopy
e. Means death of the fetus before 28
d. Methotrexate
weeks of gestation
Answer: E
One of the following is wrong regarding
All are true about recurrent abortions except
recurrent abortion:
a. incidence is 1%
a. paternal age is a risk factor
Miscarriage
b. screening for bacterial vaginosis is b. defined as 3 consecutive induced abortions c. can
useful be due to protein C deficiency
c. it is advisable to do screening for d. can be due to anti-phospholipid syndrome
toxoplasmosis Answer B
Answer : C
All of the following are true about Regarding abortion, all the following are correct except
abortions except a. Asherman’s syndrome is a recognized cause of recurrent
a. In inevitable abortions uterus is abortion
usually small for date b. In inevitable abortion, the cervix is closed
b. In incomplete abortions the cervix is c. In incomplete abortion, the cervix is open
open d. Ovulation can occur 2 weeks after abortion
c. The best method of management for e. In missed abortion, the best method of treatment is
missed abortion is medical medical termination
Answer : A Answer: b. In inevitable abortion, the cervix is closed

All of the following are true about A lactating women, 6 weeks pregnant, has mild
threatened abortion except abdominal pain and vaginal bleeding.On ultrasound,
A- Fetal heartbeat is absent there was intrauterine sac. What to do ?
B- Most cases make it to term a. reassure and ask her to come after 2 weeks
C- Most common cause of abortion b. give her methotrexate
Answer: A c. induce abortion
Answer : A
Which of the following isn’t a cause of
7 weeks amenorrhea, presented with left lower quadrant
first trimester abortions
pain and mild vaginal bleeding. Her vital signs were stable.
Answer: Cervical incompetence
On ultrasound, the uterus was empty. Her beta hCG
was 3000 mIU/ml. Your management would be:
All of the following are true about
a. single dose IV methotrexate
Recurrent abortion except ?
b. repeat beta hCG after 48 hours
Answer : Cervical incompetence is a cause
c. repeat ultrasound after one week d. immediate
of first trimester abortions
laparotomy
Answer : A
History of amenorrhea for few
weeks, presented with abdominal
Recurrent miscarriages can be due to all of the followings
pain and severe bleeding with
except:
passage of tissues. Now the bleeding
has stopped and on ultrasound the
A- Parental translocation
uterus is empty. Your management would
B- Subserous Fibroids ( Submucous is the cause NOT
be:
Subserous )
a. this is a complete abortion,
C- Double Uterus
observe for bleeding
b. repeat beta hCG after 2 days D- Hormonal dysfunction
c. this is a threatened abortion, reassure E- Asherman’s Syndrome
her
d. antibiotics & evacuation
Answer: A
Regarding abortion, which of the
following is NOT true
false about threatened abortion:
- Bacterial vaginosis in first trimester
cause preterm labor ( NOT sure ) - Answer : cervix is dilated
Miscarriage
Regarding recurrent pregnancy loses, one is false:
Which of the following is Not a bad
prognosis in threatened abortion: A- Routine cervical cerclage is indicated
B- Chromosomal abnormalities is the commonest cause
A- Bleeding at 13 week C- Early pregnancy loss is more common than mid
B- Age more than 40 trimester loss
C- Prolonged bleeding D- Antiphospholipid Syndrome is one of the most common
D- Heavy bleeding association of recurrent pregnancy loss
E- At least half of the cases are remained unexplained

Which one is false about incomplete


miscarriage ? Which of the following is Not true about miscarriage ?
A-Pt presented with colicky pain with
vaginal bleeding A- Bacterial vaginosis at FIRST trimester cause preterm
labor
B-opened cervix
B- Missed miscarriage diagnosed if 2 US weak apart
C-D&C is mandatory to be done present gestational sac more than 20 mm with no
D-b HCG is mandatory evidence of activity .

GTN
Hydatidiform mole is characterized by all of the Only one of the following is Correct regarding GTN:

following except : A- partial mole may reach term


B- partial mole is diploid
A- Hypothyroidism C- vagina is the most common site for mets.
B- Uterus larger than D- most common chromosomal karyotype in
C- Severe Hyperemesis complete mole is 46 XY
Answer A E- Embryonic tissue is part of complete mole
Answer: A
GTN
Regarding molar pregnancy, one of the following is
less likely to occur in complete mole than in partial
mole:
Complete mole associated more than partial mole in all
A- Focal rather than diffuse trophoblastic of the following except:
proliferation
B- Theca lutein cyst A- complete mole ass. with hyperthyroidism
C- Hyperthyroidism B- focal rather diffuse trophoblast proliferation
D- Uterus Large for gestational age Answer B
Answer: Focal rather than diffuse trophoblastic
proliferation

About molar pregnancy, all true except:


False about GTN:
Answer : Theca lutein cyst is always present Answer : patient can take OCPs before hCG is normal

Hydatidiform mole is characterized by all of the


following except:
a. Hypothyroidism
Compared to partial mole, all are features b. Uterus larger than
of complete mole except: date c. Severe
a. focal trophoblastic proliferation Hyperemesis
b. usually presents with large-for-date uteru d. Theca lutein cysts of the
Answer : A ovary e. Early onset
preeclamsia
Answer: A

Regarding hydatiform mole, all the following


are correct except Regarding hydatidiform mole, one is true:
a. Occurs once in 1200-1500 pregnancies in a. Preeclampsia in the most common symptom and it
the west occurs in 70% of patients
b. Has only maternal chromosome in b. Fetal parts are present
complete mole c. May present with a small c. Diagnosis is confirmed with very low levels of
for date uterus human β gonadotropin
d. May be complicated with pulmonary d. Risk of malignancy is 15-20%
embolus after evacuation e. Prophylactic chemotherapy should be given
e. Complete mole is best diagnosed by ultrasound Answer: D
Answer: b. Has only maternal chromosome in
complete mole
which statement is incorrect regarding GTN:
23) About GTN, all are true except:
a. myelosuppresion is an uncommon side A- complete moles are diploid with 46XX
effect of methotrexate karyotype
b. most common site of metastasis is the lungs B- in complete mole uterus is large for GA
c. partial mole never makes it to term C- associated with hyperemesis gravidarum
d. best treatment is suction and curettage D- plateau in B hCG levels raise suspission for
e. most common karyotype of complete mole is malignancy
46XX E- partial mole present with snowstorm
Answer : C appearance on US

Regarding Molar pregnancy, one of the following


What is the most common site of mets in GTN ?
is false?
Answer : Lunges
A- History of molar is a risk factor
GTN
B- IUCD is the preferred mode of
contraception after evacuation
C- Uterus is usually large for date
D- Exaggerated hyperemesis gravidarum is
expected
E- Complete molar pregnancy is usually
diploid

RH isoimmunization
All of the following patients must be given anti-D
Which one is not an indication to give anti D : in an Rh-negative female with an Rh-positive
husband except
A- Antepartum hemorrhage at 33 wks
B- Complete abortion at 19 wks A- Threatened abortion at 6 weeks
C- External cephalic version at 37 wks B- Ectopic pregnancy
D- Threatened abortion> 7 week C- Amniocentesis
A:D D- External cephalic version
Answer: Threatened abortion at 6 weeks
RH isoimmunization
All are true about RH- isoimmunization, except: Features of transfused blood for fetus for Rh-
sensitized woman include all of the following
A- Intrauterine blood transfusion is one of the
treatment options except:
B- Fetal hydrops is a recognized fetal complication
C- Middle cerebral artery Doppler is used in the A- low PCV
follow up of affected fetus B- O negative
D- Unlikely to happen in the first pregnancy C- compatible for the fetus and the mother
E- Anti- D should be given to all sensitized patients D- irradiated
Answer: E E- tested for infections

Regarding rhesus disease, only one of the


28. case of the need for a blood transfusion with the following is correct
inability to cross-match. Units of a. Is a cause of first trimester
which of the following blood groups must be miscarriage
administered
b. Is a cause of hydrops fetalis
a. O Rh negative
c. Occurs when the Rh status for both partners
b. A Rh negative
is negative
c. A Rh positive
Answer: O Rh negative d. Is associated with higher risk of fetal
anomalies
e. Anti-D immunoglobulin is IgM
Answer: b. Is a cause of hydrops fetalis
All are indications to give anti-D except: All of the following are presentation of RH –
a. amniocentesis isoimmunization except:
b.complete spontaneous abortion at 7 weeks gestation
c. non-sensitized Rh-ve women whose partner is Rh+ve A- fetal death
d. abdominal trauma B- Polyhydramnios
e. external cephalic version C- Enlarge fetal heart
Answer : B D- Post term pregnancy
E- Abnormally CTG
Not a presentation of RH isoimmunization :
Which of the following is NOT an indication for anti D:
A- Oligohydraminos
B- IUFD Answer : Spontaneous miscarriage at GA 7 weeks
C- Decrease fetal activity
PROM and pre-term delivery
The most important risk factor for preterm labor:
False about Preterm labor :
A- previous preterm
A- Erythromycin indicated in PROM B- cervical incompetence
B- NSAID not recommended after 32 C- premature rupture of membrane
C- Weekly steroids between 28-34 D- Bacterial vaginosis.
Answer : C E- Age < 19
Answer: previous preterm

All of the following are true regarding Preterm labor


All are indication of delivery in preterm premature
except:
rupture of membrane except:
A- Complicates 50% of pregnancies
B- Home uterine activity monitoring was A- elevated maternal WBC.
proved to decrease the risk B- fetal distress.
C- Positive fetal fibronectin indicates high risk C- chorioamnionitis.
for preterm delivery D- reaching term.
D- Previous preterm is a strong risk factor E- abruption of placenta
Answer: Home uterine activity monitoring was Answer : A
proved to decrease the risk

All of the following are risk factors for preterm


labour except:

A- Bacterial vaginosis All of the following are indications for delivery in


B- Age >35 preterm premature rupture of
C- Anemia membrane, except:
D- white race a. Twin pregnancy
E- Smoking b. Severe abruptio placenta
F- Meconium c. Fetal distress
G- antepartum hemorrhage d. Chorioamnionitis
Answer: D & F ( the question was in 2 forms in e. Reaching 37 weeks gestation
Answer: E
different answers , other choices 100% are risk
factor )

The main adverse outcome associated with urinary Indication for immediate delivery of PPROM except
tract infection is
A- Intrauterine growth A- dead fetus
restriction B- placental abruption
B- Intrauterine fetal
death
C- Septicemia
PROM and pre-term delivery
D- Anemia
E- Preterm labor

A 38 week pregnant female presents with


Delivery in a patient with PPROM is indicated in all of
abdominal pain and moderate vaginal
the following except
bleeding. Her cervix is 5 cm dilated. CTG shows
mild contractions and obstetric US is
normal. What is the next best step of A-Malpresentation
management?
B-Chorioamnionitis
a. Emergency CS
b. AROM C-Reached term
+oxytocin D-Fetal distress
c. Prostaglandins
d. Expectant management
Answer: Malpresentation
Answer:
24 years old female, primi at 37 weeks
gestation, presented with regular uterine
contractions every 3 minutes. Her vital signs
were normal, and her CTG was reactive. What
is your management:
A- allow her to deliver
spontaneously
B- ARM & oxytocin
C- vaginal prostaglandin
D- tocolytic
E- steroids
Prolonged labor & Induction of labor
Contraindications for induction of labor include all
All of the following are contraindication of induction
of the following except
of labor except:
A- Pre-eclampsia
A- abnormal CTG
B- Abnormal antenatal CTG
B- breech presentation
C- Transverse lie
C- active labor
D- Previous classical C/S section
D- vaginal prolapse
E- Triplets
E- antepartum hemorrhage
Answer: Pre-eclampsia
Answer: D
About induction of labor, one is false:
Complications of post maturity include AII the
A- The use of PG is preferred when the cervix is
following except unfavorable
B- Active genital herpes infection is a
A- Prolonged labor contraindication
B- Oligohydramnios C- The use of cervical ripening balloon is
C- Intrauterine fetal death contraindicated in patients with lower
D- Respiratory distress syndrome segment C/S
E- Meconium aspiration syndrome D- Failed induction is more is primigravida
Answer: Respiratory distress syndrome E- Prolonged pregnancy is the most common
cause
All are significant findings in 40 weeks
gestation except:
A- epigastric Complications of postdate include all of the
following except:
pain
a. Respiratory distress
B- vaginal
syndrome
spotting b. Fetal hypoxia
C- lower c. Shoulder
limb dystocia d.
edema Oligohydromnios
D- blurred e. Meconium aspiration
vision
Answer: A
E- persistent
vomiting
Induction of labor is indicated in all of the
following except :
a. Diabetes mellitus.
b. Abnormal antenatal All of the following statements are true about
CTG. induction of labor except
c. Chorioamnionitis. a. Post-date pregnancy is the most common
indication for induction
d. Intrauterine fetal
b. It increases the risks of cesarean section
death
c. Indicated in 35% of pregnancies
e. Growth
restriction.
Answer: B
Prolonged labor & Induction of labor
Contraindications for induction of labor include
all of the following except Not an indication for induction of labor:
a. Post-date a. preeclampsia at 39 weeks
pregnancy b. placental abruption at 37
b. Cervical weeks c. fetal growth
carcinoma restriction
c. Abnormal antenatal CTG d. 36 weeks gestation who has well
d. Previous classical Caesarean section controlled DM
e. Triplets

Multiple pregnancy
Regarding twin delivery, which is false:

A- episiotomy is not routinely performed


B- Continuous fetal heart rate monitoring is
recommended Regarding dizygotic twin all true except:
C- Epidural analgesia can be safely used
D- If the second fetus becomes tansverse in A- always dichorionic
lie, an emergency C/S should be B- can be concordant or discordant gender
performed C- it is a result of one egg fertilized by two
E- Augmentation with oxytocin if needed is sperms
not contraindicated Answer: C
Answer: If the second fetus becomes transverse in
lie, an emergency C/S should be Performed

Regarding multiple pregnancy, one of the


following is true:

A- The rate of mono-zygotic twins is racial


dependent
B- Chronicity is best determined at 7 weeks
Which of the following statements is incorrect about
GA
monozygotic twin pregnancies?
C- The rate of structural abnormalities is less
a. They can be of the same or opposite sex
than in singleton
D- Monozygotic twins are more common
b. The most common type is monochorionic diamnioti
than dizygotic twins
E- In monozygotic twins, the risk of Down
syndrome is the same for both fetuses
Answer: Chronicity is best determined at 7 weeks
GA or E !!

multiple pregnancies , time of Cleavage is after 3


Regarding multiple pregnancies , which of the
days , what’s the Nature of Membrane ?
following is true ?
A- monochorionic diamniotic
Multiple pregnancy
B- Diamniotic, dichorionic A- routine cervical circuits doesn't decreases the
C- Mono amniotic, monochorionic risk for preterm labor
D- Conjoined twins B- after 3 days = dichorionic diamniotic

regarding multiple pregnancies, which is false?

A- Increased risk of hyperemesis gravidarum


Regarding twin twin transfusion syndrome which of
B- Increased risk of preeclampsia
the following is false?
significantly
A-Recipient is oligo
C- Prematurity is the main cause of perinatal
B-Donor is anemic
mortality
C-Occur in dichorionic diamniotic
D- High order multiple means two fetuses or
D-Occur in 2nd trimeste
more
E- Routine cervical cerclage doesn’t reduce
the risk of prematurity in twin
Twin pregnancy may increase all of the following
complications, except:
Twin delivery which is false
a. DM
Answer : it the second baby is in transverse lie
b. preeclampsia
emergency CS should be preformed
c. post maturity
d. malpresentation
About multiple pregnancy, one is wrong:
About delivery in multiple pregnancy, one is a. all dizygotic are diamniotic
wrong b. conjoint twins occur if separation of the zygote
a. cephalic presentation of 1st twin and happens after 12 days
transverse of 2nd is an indication for C section c. twin-twin transfusion is a recognized
b. episiotomy is not routinely done complication in monochorionic
c. oxytocin augmentation is not contraindicated twins
d. continuous CTG is recommended for both twins d. best time to diagnose chronicity is 16 weeks
gestation
Regarding dizygotic twins, all are true
Cord entanglement and intrauterine fetal death
except:
occur most in which of the following:
a. Have different genetic factors
a. Conjoined twins
b. Constant incidence
b. Monochorionic mono
worldwide
amniotic
c. Same or opposite sex
c. Monochorionic diamniotic
d. Results from fertilization of two ova by
d. Footling breech
two sperms
e. Dichorionic diamniotic
e. Have double or fused placenta
Splitting of a single fertilized ovum between 8-12
days results in:
a. Conjoined twins
b. Monochorionic diamniotic
twin
c. Dichorionic diamniotic
d. Dizygotic twin
e. Monochorionic mono amniotic
Pre-eclampsia & eclampsia
Which of the following is the best
reason for the treatment of high blood
pressure
in pregnancy?
A- Prevent maternal
complications such as stroke
All are indications for delivery in preeclampsia except:
B- Lower the incidence of pre-
A- Frontal headache
eclampsia
B- Flashes of light
C- Lower the incidence of
C- Renal failure
oligohydramnios
D- Persistent vomiting
D- Lower the incidence of
A- Decrease liquor “Oligohydramnios”
intrauterine demise
Answer: E
E- Incidence of IUGR
Answer: Prevent maternal
complications such as stroke

All can be used in sever PET except : Not preeclampsia complication:-


A- Meythel dopa A- hemoconcentrate
B- Diazepam B- decrees plasma volume
C- Hydrazaline C- Increase renal perfusion
Answer: C
IV Hydralazine in management of pre-
All of the following risk factor for preeclampsia except:
eclampsia, cause all except:
A- Smoking
A- Increase in co
B- First question
B- Increase in heart rate
C- extreme ages
C- Increase in PVR
D- Previous preeclampsia
D- Decrease mean arterial
E- Age < 19 years
pressure
F- nulliparity
E- Facial flushing
Answer: A
ANSWER: C

All of the following indicate severe pre-


eclampsia except
Definite management of preeclampsia :
A- Pulmonary edema
A- Pregnancy termination
B- Oligohydramnios
B- Mgso4
C- Headache and neurological
symptoms
D- Renal failure
Answer: Oligohydramnios

One of the following is not associated


Which of the scenarios is a patient with severe pre-eclampsia
with pre-eclampsia:
A- Patient with BP 150/100 and +1 proteinuria
A- Thrombocytopenia
B- Patient with BP 140/100 and +2 proteinuria
B- Hemo concentration
C- Patient with BP 140/100, +1 proteinuria and severe
C- Increased renal perfusion
headache
D- Reduced intravascular volume
D- Patient with BP 150/100 and 2g in 24 hour urine
E- Increased risk of thrombosis
collection
Answer: Increased renal perfusion
Pre-eclampsia & eclampsia
Answer: Patient with BP 140/100, +1 proteinuria and severe
headache

Risk factor for PET except: Management in severe PET one is correct:
Answer : Low social economics ( not A-methydopa for stabilization
sure ) B-delivery regardless gestational age
False about PET:
All of the following used in
A- Spinal anesthesia is the best
management in HTN except
B- Postpartum iuresis is good sign
C- Should be monitored Postpartum for possibility of
Answer : furosemide
recurrence

Definite Rx of preeclampsia
What is the Drug of choice for Preeclampsia :
A- Pregnancy termination
B- Mgso4
- Answer: Mg sulfate
In preeclampsia, all of the following
are indications for delivery except: All are maternal complications of PET except:
a. sub capsular hematoma
a. BP 160/115
b. renal failure
b. platelet 100,000
c. fetal growth restriction
c. HELLP syndrome
d. DIC
d. neurological symptoms
e. abruptio placenta
e. proteins >5g/24 hr
Treatment of hypertension in All of the following should be routinely monitored during
pregnancy is to avoid: magnesium sulphate infusion, except:
a. IUGR
a. Blood pressure
b. IUFD
b. Respiratory rate
c. oligohydramnios
c. Urine output
d. placental abruption
e. maternal stroke d. Pupil size (pupillary reflex)
e. Deep tendon reflexes
All of the following are true
about MgSO4 except In eclampsia, which is true ?
a. Can lead to heart conduction a. Diuretic drugs are essential in all case
problems b. Always occur intrapartum.
b. Can be used safely in c. Urinary output is increase
patients with heart diseases d. Ergometrine should be avoided in the third stage of
c. Is given in higher doses to labor.
treat eclampsia e. Cesarean section must be carried out in all cases.

In a 17 weeks pregnant female with hypertension, all of the


following can be causes for her hypertension except
a. Pre-eclampsia
b. Thrombophilia
c. Cushing’s syndrome
d. Molar pregnancy
e. Systemic Lupus Erythematous
Pre-eclampsia & eclampsia
Which of the following is not
considered severe preeclampsia?
In eclampsia, which is true ?
A- BP 140/100, +2 Protein with
A-Diuretic drugs are essential in all cases.
IUGR baby
B- Always occur intrapartum.
B- BP 150/100, +1 Protein with
C-Urinary output is increased.
blurred vision
D-Ergometrine should be avoided in the third stage of labor.
C- BP 160/120, +3 Protein
E-Cesarean section must be carried out in all cases.
D- BP 140/90, +2 Protein with
headache
E- BP 150/100, +2 Protein
All of the following are Sign of severe
preeclampsia except?
A- Placental abruption
B- Polyhydroamnios Preeclampsia increases the risk of all the following except:
a. IUGR
C- Intrauterine growth restriction
b. polyhydramnios
D- Intracranial bleeding
A- Renal failure
Infection during pregnancy
All can be transmitted by vaginal delivery except :
All are signs / symptoms occur in
A- HIV
chorioamnionitis except:
B- HSV2
A- Tender abdomen
C- toxoplasmosis
B- Hypotension
D- Chlymdia
C- Tachycardia
E- Gonorrhea
D- Bad CTG
E- Fever
Answer : C
Answer: D or B
Regarding intrauterine infection, which of the following is
false:
A- Intrauterine infection is an important cause of Not a sign of
brain injury chorioamnionitis :
B- May result from hematogenous transplacental A- Fever
spread secondary to maternal bacteremia B- Tachycardia
C- May result in periventricular leukomalacia (PVL) C- Abdominal
D- Does not occur in the presence of intact membrane tenderness
E- Chorioamnionitis is an indication for induction of D- Hyper-reflexia
labor Answer: Hyper-reflexia
Answer: Does not occur in the presence of intact
membrane
Earliest sign of chorioamnionitis:
A- maternal tachycardia
B- fever
C- abdominal tenderness
D- vaginal bleeding

DM
All the following occur in GDM except : All of the following are performed at 13 weeks in a chronic DM
A- Hypocalcemia patient except:
DM
B- cardiac anomalies A- OGTT
C- caudal regression syndrome B- Rubella IgG and IgM
D- respiratory distress C- CBC
syndrome D- Urine culture
E- hypoplesia of islets of E- Obstetric US
pancreas Answer: A
A:e

Regarding a pregnant patient with DM,


one of the following is false:
A- Glucosuria is a reliable
parameter for control
In a pregnant with type 1 diabetes all of the following are true
B- Glycosilated Hb is best kept
except
below 6
A- Could present with polyhydramnios
C- DKA is more common in
B- They are at increased risk of shoulder dystocia
pregnant than non-pregnant
C- They are at increased risk of aneuploidy
D- Insulin requirement increases
D- Increased risk of IUFD
as pregnancy advances
Answer: C (most likely)
E- Polyhydramnios is a common
complication
Answer: Glucosuria is a reliable
parameter for control
About diabetes in pregnancy, which
one is wrong
a. well controlled DM can develop
One of the following is true about diabetes in pregnancy:
macrosomia
a. most diabetic patients present as hypertension
b. fetal death may occur in well
b. +babies of controlled diabetic mothers have nearly same
controlled
complications as non-diabetic
DM
c. congenital anomalies increase in mothers
gestational DM

About diabetes in pregnancy, one is


correct: Fetal hyperinsulinemia causes increased growth of the
a. diet control alone increases the following fetal tissue, except:
risk of developing hypertension a. Brain
b. normal blood sugar at 20 b. Liver
weeks rules out the c. Heart
development of DM d. Muscles
c. insulin requirement increases in e. Subcutaneous fat
puerperium
Which of the following is true
about diabetes in pregnancy The following conditions may be associated with
a. Risk of polyhydramnios is diabetes, except:
lower if it diabetes is controlled a. Hypertension
b. OGTT should be performed if b. Post maturity
glycosuria occur before 16 weeks of c. Fetal encephalocele
gestational age d. Ventricular septal defect
c. The best way to control diabetes is e. Fetal anal atresia
with two doses of soluble insulin
DM
d. During labor serum glucose must be
maintained between 6-8 mmol/L
e. If the patient’s HbA1c is 11%, the
pregnancy must be aborted
Regarding diabetes with pregnancy,
only one of the following statement is
true:
a. Strictly controlled diabetes will
prevent development of fetal
macrosomia.
All of the following are associated with Type I diabetes
b. Sudden fetal death may occur mellitus in pregnancy except
with well controlled diabetes. a. Trisomy 21
c. Excessive fetal growth usually
b. Polyhydramnios
starts around the 20th week of
c. Large baby
gestation.
d. Congenital anomalies
d. Neonatal morbidity among babies of
diabetic mother is around 40%.
e. Risk of congenital
malformations is significantly
increased in
gestational diabetes.
Which of the following is the best test to screen for
Which of the following is true
gestational diabetes at 24-28 weeks-of gestational age
about gestational diabetes
a. Fasting blood glucose
mellitus
b.HbA1c
Answer: It increases the future risk
c. OGTT
of developing diabetes mellitus
d. 1 hour test after 50 g glucose challenge
Which of the following is true about
the infant of a diabetic mother Which of the following is true about GDM ?
a. There is a risk of development of A. you need to check dose of insulin with progress of pregnancy
neonatal hypoglycemia and B. pre-pregnancy dm is the most common
hypercalcemia
b. Large for gestational age
All of them is Complication of GDM except:
Regarding clinical diabetes in pregnancy one of the following is
A. Miscarriage
false:
B. Polyhydroamnio
C. preterm labor
- Answer: well controlled diabetes will certainly prevent IUFD
D. macrosomia

Which of the following is true regarding


diabetes in pregnancy:

A. Retinopathy risk is increasing when


correcting glucose level rapidly
B. It is associated with increased Not risk factor for gestational DM
incidence of Post-term delivery unknown cause of polyhydrominous
C. Insulin dose is progressively
decreased during pregnancy
D. Decreased fetal movement after 32
weeks is an indication for early
delivery
DM
E. A trial labor is considered if fetal
weight is more than 4.2Kg

Anemia , venous thromboembolism and bleeding disorders


Most common type of geastational anemia :
True about Venous thromboembolism:
The answer is : IDA
Answer : Warfarin execrated in breast milk !!
A 32 weeks pregnant female has a Hb of 9. The best
Regarding DVT, which one is correct :
initial management is:
A- Duplex US can't detect all cases
A- Give blood transfusion
B- its a clinical diagnosis
B- Start intramuscular iron
C- risk in pregnancy more than puerperium
C- Give IV iron
Answer A
D- No action needed
Anemia , venous thromboembolism and bleeding disorders
E- Treat with oral iron and vitamins
Answer: Treat with oral iron and vitamins

All of the following are true about


antiphospholipid syndrome except
Correct about Gestational thrombocytopenia : A- Thrombocytopenia can develop in this
A- Less than 20,000 disease
B- Less than 20000 B- Arterial and venous thrombi can occur
C- Fetus affected C- It is a congenital thrombophilia
Answer: it is a congenital thrombophilia

Regarding management of coagulation disorders


Regarding thrombocytopenia during pregnancy, which
in pregnancy, one of the following is
is true:
true:
A- HIV can cause thrombocytopenia during
A- Heparin should be stopped 7 days before
pregnancy
performing C/S safely
B- 20,000 platelet count is safe for regional
B- Osteoporosis is more common in LMWH
anesthesia
than in HMWH
C- ITP presents only in first trimester
C- Activated Protein C resistance is an
D- Fetal blood sampling is advised in all cases of
example of inherited thrombophilia
thrombocytopenia in pregnancy
D- Warfarin can be used safely during
E- Gestational thrombocytopenia carries a
pregnancy
significant pathological effect on the fetus
E- Heparin will cause thrombocytosis if used
Answer: HIV can cause thrombocytopenia during
during pregnancy
pregnancy
Answer : C
Regarding antiphospholipid syndrome, all are
correct except:
A- two positive antiphospholipid antibody
All of the following are correct except:
tests (6 weeks apart) are needed
A- CS is contraindicated if platelets 90000
fordiagnosis.
B- CMV infection in pregnancy can cause
B- Aspirin and Heparin are needed to
thrombocytopenia
overcome the potential complications.
C- IVIG can be used to treat thrombocytopenia
C- Previous one IUFD at 34 weeks is an
D- Gestational thrombocytopenia occur during
enough clinical presentation to
3rd trimester
confirmthe diagnosis.
E- Platelets transfusion is indicated if count is less
D- one previous missed miscarriage is an
than 50.000 (or somethinglike this)
enough clinical presentation toconfirm
ANSWER: A
the diagnosis.
E- It's a risk factor for DVT.
Answer: 2 options can be the answer, A or D
Regarding anticoagulants in pregnancy. All are correct
except: All are true regarding DIC except
A- Warfarin may cause chondrodysplasia A- it's a recognized complication of
punctata. prolonged missed miscarriage.
B- Prophylactic LMWH should be stopped about B- FDP will drop.
12 hours before delivery. C- platelet count will drop.
C- LMWH is a risk factor for osteoporosis. D- multidisciplinary team work is needed to
D- Warfarin and heparin are not secreted in milk. manage DIC.
E- LMWH can cause thrompocytopenia more E- PT and PTT will prolong.
than unfractionated heparin. Answer: B
Answer: E
Anemia , venous thromboembolism and bleeding disorders
Regarding thromboembolic disease in pregnancy.
Regarding thrombophila and pregnancy, All the all are correct except:
following are correct except: A- preeclampsia is a risk factor for DVT.
A- Protein S deficiency is a congenital B- BMI more than 30 is a significant risk
thrombophila. factor for DVT
B- increase the risk of post-maturity. C- most DVT are left sided
C- increase the risk of fetal death in third D- puerperium has lower risk for
trimester. thromboembolism than during
D- its adverse effect increase with maternal age. pregnancyitself.
E- increased risk of severe abruption. E- nearly half of DVT patients are
Answer: B asymptomatic.
Answer:D

Regarding thrombocytopenia in pregnancy, All are


correct except:
A- Gestational thrombocytopenia usually occur in
the 3rd trimester. About iron deficiency anemia in pregnancy, one
B- CMV infection is a recognized cause of is wrong:
thrombocytopenia in pregnancy. a. ferretin is low
C- Cesarean section is contraindicated at 90,000 b. +baseline management is IV iron c. multiparity
platelet count. increases the risk
D- IVIG can be used to increase platelet count.
E- vaginal delivery is preferred over Cesarean
section.
Answer: C

Regarding folate deficiency anemia in pregnancy, all About folate deficiency anemia, which is
of the following are risk factor wrong:
except : a. +using anti-convulsant therapy reduces the
a. Anticonvulsant need for folate
therapy. b. Thalassemia b. MCV increases
trait. c. hyper-segmented neutrophils on
c. Meckel gruber blood film d. second most common
syndrome. d. Haemolytic anemia
anemia. e. folate supplements lower the incidence of
e. Hereditary spherocytosis. neural tube defects
Answer : C
About DVT, all are true except:
a. more on the left side
b. +heparin causes cardiac defect
c. in a patient with history of DVT, treatment
should continue 6 weeks postpartum

Regarding coagulation disorders in pregnancy,


Which of the following statements is true
one statement is true:
about coagulation disorders in pregnancy
a. Treatment of proven DVT in pregnancy should be
a. DVT is more common in pregnancy than in
started with
puerperium
heparin and then shifted to warfarin
b. Warfarin is contraindicated while lactating
b. Superficial thrombophlebitis is treated by bed
c. Patients with DVT are treated with heparin
rest, analgesia and local heat
and then warfarin
c. Warfarin is excreted in breast milk
Anemia , venous thromboembolism and bleeding disorders
d. Heparin crosses the placenta and should be d. 50% of DVTs are
avoided in the 1st trimester asymptomatic
e. D-dimer test is important in the diagnosis of
DVT in pregnancy
Answer: B
All of the following are true about antiphospholipid
syndrome
except
a. Thrombocytopenia can develop in this disease
b. Arterial and venous thrombi can
occur c. It is a congenital
thrombophilia
Answer: it is a congenital thrombophilia

Others medical disorders


About hyper-emesis gravidarum, all are true except:
Which is wrong about cholestasis ?
A- can be associated with hyperthyroidism
A- Dx by exclusion
B- can cause Wernicke's encephalopathy in severe
B- Air embolism is complication
cases
C- Itchy (Palm sole )
C- more with multiple pregnancy
A:B
D- usually resolves by 12 weeks
E- Associated with HTN
Regarding abdominal pain in pregnancy, which
of the following is false:
A- Gallbladder stones can cause
All are true regarding cholestatic disease of pregnancy
pancreatitis
except:
B- Appendicitis is the most common
A- Itching is the most common complaint
cause of acute abdomen
B- Abnormal liver function is diagnostic
C- complicated ovarian cyst need
C- Leads to meconium stained liquor
immediate intervention
D- High rate of recurrence
D- Peptic ulcer ALWAYS improve in
E- No skin lesions
pregnancy other forms ( Peptic ulcer
Answer: B
usually deteriorates in pregnancy )
E- Abruptio placenta can present with
abdominal pain only.
Answer D
Regarding Acute fatty liver disease, which one is false:
all are false except: A- Disease of third trimester
A- Itching is mostly in the abdomen B- Treatment in intensive care setting and delivery
B- Leads to preterm labor C- Parity is not a risk factor
C- Commonly occurs in the first trimester D- Recurrence is 15%
D- Leads to acute fatty liver E- Hyperglycemia
Answer: B
ANSWER: E
Others medical disorders
Regarding asthma in pregnancy, all are correct except:
Regarding fatty liver in pregnancy, which is A- increased risk of SGA
wrong: B- steroids are contraindicated in pregnancy.
A- Rx is ICU and delivery C- some patients improved during pregnancy.
B- They have hypoglycemia D- status asthmaticus is treated as non-pregnant
C- Parity is not an indicator woman.
Answer: Both B and C seem to be correct E- Delivery by CS depends on obstetric indications.
Answer: B
About hyper-emesis gravidarum, all are true
except: About cholestasis, one is false:
a. polyuria is a sign of severity a. +may lead to amniotic fluid embolism
b. can be associated with hyperthyroidism b. it's a diagnosis of exclusion
c. can cause Wernicke's encephalopathy in c. leads to itching in mainly palms and soles d.
severe cases associated with fetal distress
d. more with multiple pregnancy e. associated with preterm labor
e. usually resolves by 12 weeks

Regarding epilepsy in pregnancy, all of the All are true about thyroid disease in pregnancy
following statement are true except : except:
a. Monotherapy is the gold standard. a. +graves' disease worsens in the 3rd trimester
b. Breast feeding is not contraindication. b. hyperthyroidism is associated with stillbirth c.
c. Risk of congenital abnormality is around 3%. propylthiouracil is safe in lactation
d. 1/3 of the patients will experience an d. thyroxine is safe in pregnancy
increase in seizure frequency. e. radio-iodine scan is contraindicated in pregnancy
e. Folic acid supplement is indicated.
Answer: B

Drugs in pregnancy , tocolytics , analgesia and anesthesia


all are contraindicated in pregnancy except: treatment of syphilis in pregnancy :
a-warfarin a- Trimethoprim
b-LMWH b- Gentamycin
c-Ciprofloxacillin c- Erythromycin
d-phenytoin d- Penicillin
Answer : B E- tetracycline
Answer : d
Regarding drugs in pregnancy,
which of the following is true:
One of the following is not a contraindication for tocolytics:
a. Rubella vaccine can be given in 3rd
a. Placental abruption
trimester
b. Transverse lie
b. Tetracycline can cause hemolytic
c. Chorioamnionitis
anemia of newborn
d. Labour with 7cm dilation
c. Tetanus vaccine is safe in 3rd
e. Fetal distress
trimester
Answer: Transverse lie
d. Cephalosporins are classified as
category A
Drugs in pregnancy , tocolytics , analgesia and anesthesia
e. The most common congenital
anomaly associated with phenytoin
use is cardiac defect.
Answer: Tetanus vaccine is safe in
3rd trimester
All of the following are
One of the following is not a tocolytic: contraindications for tocolytics
a. beta blocker except:
b. nifedipine a. Active labor
c. MgSO4 b. Fetal abnormalities
d. atosiban c. Antepartum hemorrhage
e. indomethacin d. Fetal compromise
Answer: beta blocker e. Chorioamnionitis
Answer: C
False about tocolytic drugs :
Not teratogenic drug?
A-indomethacin is the best choice to treat preterm labor with
A.Heparin
oligohydramnios
B.Warfarin
B-ritodrine cause more side effects than nifedipine
C. Phenytoin
C- there is no evidence to treat PPROM by tocolytic
D. Ciprofloxacin
D-steroid increase the risk of maternal hyperglycemia and
E. Lithium carbonate
infections
ANSWER: A
Answer: A
Regarding continuous epidural
anesthesia, one of the following is
true:
a. It is contraindicated in woman
Epidural anesthesia when compared with regional anesthesia one
with past history of DVT
of the following is false :-
b. Leakage of CSF is a recognized
b- causes less post surgery nausea
cause of headache
c- causes less bleeding
c. Hypertension treated with
d- rapid recovery after surgery
labetalol is a recognized
e-Increase Risk of DVT
contraindication
Answer E
d. It is contraindicated in suspected
IUGR
Answer: Leakage of CSF is a
recognized cause of headache
All of the following are used for pain
All of the following are true about epidural anesthesia except: relief in labor
A.It lengthens the second stage of labor except :
B It is recommended in cardiac patients a- Spinal
C.Epidural anesthesia acts faster than spinal l b- Epidural
D.Headache is a recognized complication of epidurals c- Narcotics
E. can cause meningitis d- Nitrous oxide
ANSWER: C e-Diazepam
A:e
All of the following are contraindications for tocolysis except
a. Transverse lie
b. A women in labor with 7 cm dilation c. Abruptio
placenta
d. Chorioamnionitis
Drugs in pregnancy , tocolytics , analgesia and anesthesia
Answer: Transverse lie

Which of the following vitamins should not be given in pregnancy


a. Vitamin A
b. Vitamin B12
c. Vitamin B3
d. Vitamin C

Answer: Vitamin A

Epidural anesthesia is recommended in all the following conditions


except:
a. Delivery of twins.
b. Complete breach delivery.
c. Maternal cardiac disease.
d. Platelet count less than 80,000
e. Primigravida in labor.
Answer: D

16. Which of the following is true about epidural analgesia in


labor?
a. It is contraindicated in severe IUGR
b. It is contraindicated in females with a previous history of
DVT
c. Induced hypotension can cause fetal heart changes
d. Should not be administered during the second stage of labour
e. Associated with an increase in rate of cesarean delivery
Local anesthesia is better than
general anesthesia in
Answer: it has a lower risk of deep
vein thrombosis
PG1, presented at 37 weeks in labor, CBC: Hb 10 g/dL, WBCs
5*10^9 /L, platelets About epidural analgesia, one is
100000/ μL, she’s not known to have any medical illnesses, which wrong:
statement is true: a. +prolongs 1st stage of labor
a. Epidural anesthesia is safe in this patient b. prolongs 2nd stage of labor
b. Platelet transfusion is required if platelets <80000 / μL c. CTG changes are mainly due to
c. The most likely cause is ITP maternal hypotension
d. She has pancytopenia and should receive prednisolone e. The d. increases risk of instrumental
patient should be delivered by cesarean section delivery
Answer: A e. doesn't increase the risk of CS

About drugs in pregnancy, all are true except: About epidural analgesia,
a. ampicillin is relatively safe
one is wrong
b. cephalosporin is relatively safe
Answer : can be used in a
c. +tetracycline is safe in 2nd trimester
hypotensive patient
d. phenytoin is associated with facial clefting
Drugs in pregnancy , tocolytics , analgesia and anesthesia
All of the following drugs are
matched correctly with their side
Which of the following should always be stopped during
effects except:
pregnancy:
a. Hydralazine …. Bradycardia
a.Metoclopromide b.
b. Magnesium sulphate ….
Metformin
Respiratory depression
c. Tricyclic antidepressant d.
c. Methyl dopa …..
Isotretenoin
postpartum depression
e. Propanalol
d. Nifedipine ….. headache
Answer: D
e. Indomethacin ….. premature
closure of ductus arteriosus
Answer: A

Postpartum hemorrhage
Which one of the following true about tt secondary
postpartum hemorrhage(something like that ) :-
A- Ergometrine
B- steroid is the first tt to start with it
C- starts 24hr after delivery to 6 weeks
Wrong about PPH :
postpartum
Answer : vaginal bleeding more than 800ml
D- doesn't occur after CA

Answer: C

All of the following can be used in controlling PPH


Which one is not a cause for uterine atony :
except:
A- previous Cs
a-PG E2
B- prolonged labor
b-PG F2a
C- multible pregnancy
c-Misoprostol
D- antepartum hemmorhage
d-Oxytocin
E- Intrauterine growth restriction.
e- Ergometrin
A:e
Answer A
primigravida complains of excessive vaginal
bleeding after delivering her child.
All of the following are risk factors for uterine atony Delivery was difficult and instrumental delivery
except: was used. Uterus is firm and is 2 cm below the
a. Augmentation of labor by oxytocin umbilicus. The most likely cause of her vaginal
b. Use of MgSO4 bleeding is
c. Breech Presentation a. Uterine inversion
d. Antepartum Hemorrhage (APH) b. Uterine rupture
e. Instrumental Delivery c. Retained products of conception
Answer: Antepartum Hemorrhage (APH) or E d. Uterine atony
e. Genital tract laceration
Answer: Genital tract laceration
Postpartum hemorrhage
All are causes of postpartum hemorrhage Patient with primary PPH, apart from resuscitation,
except: what is your
A- Meconium next step in managing her bleeding:
B- Instrumental delivery a. +uterine massage & oxytocin
C- Multiparty b. bimanual compression of the
D- antepartum hemorrhage uterus c. B lynch procedure
E- Anemia d. uterine artery embolization
Answer: A e. uterine artery ligation
A female gave birth by C-section 2 weeks ago,
placenta was delivered completely, now she
came febrile with secondary PPH. All are true All of the following are risk factors for
except: postpartum hemorrhage, except:
a. Cesarean delivery
a. can continue breastfeeding
b. most common cause is retained products of b. Uterine over
conception distention
c. best treatment is with ceftriaxone and c. Uterine fibroids
d. Previous PPH
metronidazole
e. Platelets count of 100,000
d. HVS and endocervical swab are recommended
Answer: E
e. uterus palpable 4 cm above the pubic symphysis
indicates sub-involution
Regarding control of bleeding after delivery, all
the following are correct except
a. Platelets <50000/ml is a risk factor for bleeding
b. Lower genital tract laceration may cause severe
bleeding due to edema and engorgement
Which of the following statements about uterine c. The lower uterine segment is effectively
inversion is incorrect contractile and retractile
Answer: Chronic inversion occurs after 24 hours of d. Deposition of fibrin over placental bed
delivery helps stop bleeding
e. Contraction and retraction of the muscles in
the upper segment is essential to close blood
vessels.
Answer: The lower uterine segment is
effectively contractile and retractile
Inversion of the uterus is almost always subsequent All of the following are risk factors for uterine
to atony except
a. Multiple a. High parity
pregnancy b. Instrumental
b.Polyhydramnios delivery
c. Difficult forceps delivery c.
d. Traction of the umbilical cord before placental Chorioamnionitis
separation d. Prolonged labor
e. Abruptio placenta e. Primigravida with small for date uterus
Peurperium
The most common causative agent of postpartum
mastitis is:
A woman who delivered vaginally a few days ago,
a. Neisseria
is now tearful, crying and complains of poor sleep.
b. Aerobic streptococcus
The most likely diagnosis is:
c. Staph. Aureus
a. Blues
d. Chlamydia
b. Depression
Answer: Staph. Aureus
c. Psychosis
Answer: Blues

Regarding peripartum cardiomyopathy, all are


Peripartum cardiomyopathy, all are true except: correct except:
A. More with Multiparity a- cerebral embolization is a major cause
B. More with multiple pregnancy of morbidity.
C. Need fluid restriction b- recurrence rate is high.
D. U have to shorten the second stage c- anticoagulants is may be required.
E. Epidural is preferred d- more common in multiple pregnancy.
Answer: E? e- more common in multiparous.
Answer: Might be A cause cerebral
embolization is a major cause of death

The most common presenting symptom/


complaint of endometriosis is:
a. Chronic pelvic pain
b. Infertility
Regarding peripartum cardiomyopathy, all are correct c. Chocolate cyst
except: antibiotics are always needed d. Dysmenorrhea
c. Oligomenorrhea
Answer: D

A patient had Normal vaginal delivery 1 week ago,


presented with fever andchills. all the following are
possilble causes except:
Uterus immediately after delivery is:
a- nipple cracks.
A- Boggy
b- endomyometritis.
B- Dicoid
c- Pyelonephritis.
C- Firm and smooth
d- mastitis.
Answer : C
e- Pneumonia.
Answer: A

In relation to pregnancy, all of the following may


increase during puerperium except: About puerperium, one is
a. wrong:
thromboembolism a. commonly develops polyuria
b. +diabetes b. +warfarin is a contraindication for breastfeeding
c. thyroiditis c. after 2 weeks, uterus should be at the level of
d. torsion of ovarian cyst symphysis pubis
e. psychosis d. ofloxacin is given in case of mastitis
Peurperium
primigravida female had vaginal delivery 10
days ago and now presents with foul
G2P2 had cesarean section 1 week ago, presented smelling discharge, fever and vaginal bleeding.
with fever. The differential diagnosis include all of She is hemodynamically stable. Ultrasound
the following except: a. Mastitis shows retained products of conception. Your
b. Lung next step of management is
atelectasis a. Immediate evacuation followed by
c. DVT antibiotics
d. Urinary tract b. Antibiotics for 5 days and review later
infection c. IV antibiotics followed by evacuation
e. Infected C/S wound d. Wait results of HVS culture to start antibiotics
Answer: B e. Immediate evacuation and send products for
culture and sensitivity
Answer: IV antibiotics followed by evacuation

All of the following are causes of fever and


Regarding breast feeding, all are true chills 7 days after delivery except
except: a. Reduce the risk of breast cancer. a. Nipple cracks
b. Suckling stimulate uterine involution. b.Endometritis
c. Breast engorgement causes fever within 24
c. Breast
hour after delivery if no milk pumping.
abscess
d. The predominant immunoglobulin is IgA.
d. Mastitits
e. Colostrum contains more protein than breast milk.
Answer: C e. Pyelonephritis
Answer: Nipple cracks
Which of the following statements
Regarding breastfeeding, all the following
regarding contraception in the postpartum
statements are correct except
period is correct
a. Suckling stimulates the release of oxytocin
a. COCP is a good choice during
b. Breast engorgement causes fever within 24
breastfeeding
hours after delivery
b. IUCD is contraindicated
c. Colostrum contains more protein than
c. Contraception usually starts 4-6 weeks after
breast milk
delivery
d. Reduces the risk of breast cancer
d. Progesterone only pills is contraindicated in
e. The predominant Immunoglobulin is IgA which
breastfeeding
provides protection in infants
e. Lactation only is adequate contraception
Answer: Breast engorgement causes fever within
Answer: Contraception usually starts 4-6 weeks
24 hours after delivery
after delivery

All are true about breastfeeding


Which of the following is true about post-
except:
partum contraception
a. breastfeeding is not recommended
a. Lactation only is an effective method of
contraception in HIV
b. Oral contraceptives should be started b. +breastfeeding is not recommended in HBV +ve
c. Should be started 4-6 weeks after delivery mothers
d. IUCD is contraindicated after vaginal c. lactation is inhibited by bromocriptine
delivery d. lactation is initiated postpartum by falling
level of estrogen

Regarding breast feeding, one is false:

A- It is inhibited by Bromocriptine
Peurperium
B- It is not recommended in Hep B + patients
C- It is initiated Post-Nataly in response to the
decline in Estrogen
D- Can inhibit ovulation
E- It is not recommended in HIV + patients

Antepartum hemorrhage and obstetrics emergency


All of the following have a risk of recurrence except
One of the following is not a risk factor for cord
a. Gestational diabetes
prolapse:
b. Pre-eclampsia
a. Multiple pregnancy
c. Placental abruption
b. Transverse lie
d. PPROM
c. Footling breech
e. Cord prolapse
d. Short cord
Answer: Cord prolapse
e. Polyhydramnios
Answer: Short cord
All of the following are true about cord prolapse which of the following is correct regarding Ante-
except: partum Hemorrhage :
A. Late decelerations are a common finding a) vasa previa is rupture of fetal blood vessels
B. Is a known complication of AROM b) grade 2 placenta previa is the placenta partially
C. Pushing the fetal head transvaginally is an covering the cervical os
important step in management c- sterile speculum examination is contraindicated
D. Is a common association with footling breech d- placenta previa can be diagnosed as early as 22
E. It is a common association with prematurity and weeks
multiparity e- placenta abrubtio best delivered by CS
Answer: A Answer: A

Regarding placental abruption, one is correct:


a. Nearly half of placental abruptions are in
established labor
All are essential tests in placental abruption except: b. Placental abruption never presents after 40
a-bood film weeks gestational age
b- cbc c. Smoking decreases the risk of placental
c-rh-isoimmunisatiin abruption
d-kft d. All patients with placental abruption should
e-pt receive tocolytics
A:a e. Continuous fetal heart rate monitoring is not
mandatory in established cases
Answer: Nearly half of placental abruptions are in
established labor.
G3P2 at 38 weeks GA presents with abdominal pain Regarding placenta previa, one is correct:
and bright red vaginal bleeding. a. The diagnosis of placenta previa is based on
On examination the uterus was firm and tender. She clinical evaluation
has regular uterine contractions b. Most low lying placentae diagnosed during
every 2 minutes. The patient had normal U/S at 34 detailed anomaly scan will move up to the
weeks. Which of the following is the upper segment by reaching term
most likely cause of her problem: c. X-matching one unit of blood is enough
Antepartum hemorrhage and obstetrics emergency
a. Placental Abruption d. All patients with placenta previa should be
b. Placental Accreta delivered by C/S
c. Labor e. Vaginal U/S is contraindicated in placenta previa
d. Placenta Previa Answer: Most low lying placentae diagnosed
e. Heavy show during detailed anomaly scan will move
Answer: Placental Abruption up to the upper segment by reaching term
Correct about placenta previa except:
Correct about placenta abrubtion:
Answer : Usually affect fetal well being
Diagnosed with US
All are indications for delivery in placenta previa
except
about placenta previa which is correct :
A. Active vaginal bleeding
- speculum examination is contraindicated
B. PPROM
C. Reached Term
Answer: B
Majority of women presenting with placental In a patient presented with moderate vaginal
abruption have: bleeding at 36 weeks, all of the following in favor
a. Chorioamnionitis of placenta previa except:
b. Blunt trauma to a. Tender uterus
abdomen b. History of bleeding in early
c. Preeclampsia pregnancy
d. No evident c. Previous cesarean delivery
factor e. Multiple d. Transverse lie
pregnancy e. Good fetal condition
Answer: D Answer: A
The earliest sign of placental abruption Initial assessment of patient presented with APH
is: at 36 weeks of gestation include all
a. Elevated INR of the following
b. Skin except:
ecchymosis a. Uterine
c. Hematuria contractions b.
d. Intrauterine fetal CBC
death c. Vital
e. signs d.
Thrombocytopenia CTG
Answer: E e. Vaginal examination
Answer: E
Regarding placenta previa, all of the following are A patient with placenta previa presents with
true except : mild vaginal bleeding at 32 weeks of
a. Associated with increase risk of gestational age that stopped completely. Fetal
hysterectomy. CTG is reactive. What is your plan regarding
b. Often causes sever PPH. her delivery
c. Typically presents with painless vaginal a. Plan for an elective cesarean section at
bleeding.
37 weeks
d. Vaginal ultrasound causes bleeding.
b. Only deliver by cesarean if bleeding
e. Accrete is associated with increase maternal
mortality. recurs
Answer: Answer: A
Antepartum hemorrhage and obstetrics emergency
G3P2, previous deliveries were by C section.
Now she's 36 weeks gestation presenting
with painless vaginal bleeding. After resuscitation,
83. A young patient at 33 weeks presents what is your next step:
with abdominal pain and moderate a. ultrasound to localize the
vaginal placenta
bleeding. Which of the following is the most likely b. give steroids
cause of her problem? c. vaginal exam
Answer: Placental abruption d. give steroids &
indomethacin
e. immediate C section
Answer A
Regarding placenta previa, all the following are In placental abruption, all of the following c
correct except: are correct except
a. Previous cesarean section is a risk a. The patient commonly present
factor in labor
b. Malpresentation is common b. May be associated with
c. The placenta is within 2 cm from the cervical os proteinuria
d. A placenta covering the cervical os at 20 c. Fetus is often in transverse
weeks is a major placenta previa presentation
e.
D Is a risk factor for preterm delivery. d. The main fetal risk is hypoxia
Answer: D e. DIC is one of its serious complications
Answer: Fetus is often in transverse presentation
All of the following are risk factor for C 37 weeks gestation presented with mild
placental abruption except placental abruption, cervix is 5 cm dilated, fetal
a. Pre- CTG reactive. After stabilization of the patient,
eclampsia b. what's your next step:
Smoking a. AROM and give
c. Oligohydramnios oxytocin b. emergent
d. Direct abdominal CS
trauma c. give prostaglandin
e. Chorioamnionitis d. admission and regular CTG every two days
Answer: Oligohydramnios e. steroids
Answer: A
True about cord prolapse:
a. can be diagnosed by palpation while
membranes are intact About cord prolapse one statement is false :
b. +can be delivered vaginally if
cervix was fully dilated - feeling the cord on PV with intact membranes
c. common cause of intrapartum fetal death (this is cord presentation not prolapse, prolapse
d. using vaginal PG in induction increases the has ruptured membranes)
risk
e. incidence 1 in 5,000
True about shoulder dystocia:
a. +many affected babies are of normal body
weight
b. the best management is to keep pushing &
traction
c. obstruction occurs at the pelvic outlet
d. 70% is the risk of recurrence
Antepartum hemorrhage and obstetrics emergency
e. preventable in majority of cases

Fetal malpresentation
whats false about bregma :
a-Located in posterior fontanelle which one mismatch :
False about Fetal skull : a- Face presentation…. Suboccipito- bregmatic
a- Posterioir fontanele called bregma b- Brow presentation … mento occipital
b- Slide on each other Answer A
Answer : a

You feel a fetal nose and mouth. The chin is pointing


Relationship of fetal parts to each other is
toward the maternal left pubic
called:
rami. This is a presentation of:
a-Position
a. Face, mentoanterior
b-Station
b. Occiput, transverse position
c-Lie
c. Brow presentation
d-Presentation
d. Transverse lie
e- Altitude
e. Vertex presentation
Answer : A
Regarding the presenting of fetal diameters, one One of the following is not associated with breech
of the following is a false match: presentation:
a. Sub-occipitobregmatic diameter Face a. Multiple pregnancy
presentation b. Prematurity
b. Sub-mentobregmatic diameter Hyper- c. Placenta Previa
extended fetal head d. Congenital uterine anomalies
Fetal malpresentation
c. Bi-parietal diameter (BPD) = 9.5 cm e. Full maternal bladder
d. Occipitofrontal = 11.5cm Answer: Full maternal bladder
e. Mento-vertical diameter Brow presentation
Answer: Sub-occipitobregmatic diameter Face
presentation

In cases of transverse lie, all are correct except:


a- of the second twin, CS is the treatment of
A patient 35 weeks pregnant presents to the clinic for a
choice.
followup visit. On US her fetus is in a breech position.
b- the fetal head is commonly to the mother's
Which of the
left.
following is the next best step of management?
c- there is associated placenta previa in about
a. Schedule a repeat visit in 2 weeks
10%
b. External cephalic version
d- the fetal back is usually anterior.
c. Deliver by elective cesarean section
e- there is associated cord prolapse in 15% of
Answer: Schedule a repeat visit in 2 weeks
cases.
Answer: A
ALL are cause of face presentation except: One is false:
A. Anencephaly A- submento-bregmatic ---> Hyperextended.
B. Branchial cyst B- Mentovertical ---> well flexed head
C. Multiple pregnancy C- Biparietal diameter is 10 cm
D. Multiparty D- Occipito frontal → Vertex
E. contracted pelvis E- Suboccipitobragmatic → flexed
Answer: D (most likely) Answer: B
Most common malpresentation:
Mismatch:
Answer : Complete breech
Answer : Brow-mentovertical

In vaginal examination of a pregnant lady in


The most common predisposing factor for breech
labor you feel a nose and a mouth.
presentation is:
The chin is towards the left hip of the mother.
a. Congenital uterine abnormality
What is true about the position?
b. Congenital fetal abnormality
A- mentum transverse position
c. Multiparity
B- brow presentation
d. Prematurity
C- vertex presentation
e. Placenta previa
D- transverse lie
Answer: D
E- occipitotransverse position
Answer: A

The denominator in face presentation is:


a. Occiput
One of the following is wrong: b. Chin
a. +suboccipito-bregmatic is face presentation
c. Bregma
b. brow presentation must be delivered by C
d. Nose
section
e. Anterior fontanel
Answer: B

Which of the following statements


While conducting a pelvic examination for a 38 week
regarding malpresentation and
female in labor, you feel a nose
malposition are incorrect ?
Fetal malpresentation
a. Persistent occipito-posterior position in the and mouth. The patient’s chin is pointed towards the
second stage of labor is delivered by cesarean mother’s pubic ramus. Which of the
section following is true about the presentation/position of
b. Footling breach are best delivered by the child?
cesarean section a. Face presentation- mentoanterior
c. The use of vacuum is contraindicated in face b. Persistent occipitoposterio
presentation d. Both polyhydramnios and
oligohydramnios increase the risk of breech
presentation
e. Traction forceps are used for delivery of the
after-coming head in breech presentation

Which of the following is


mismatched
a. Subocciptobregmatic – 12.5 cm
b. Occipito-mental – brow
presentation
c. Occipito-frontal --- 11.5 cm
d. Submentobregmatic – Face presentation
Answer: Suboccipitobregmatic – 12.5 cm
(should be 9.5 cm)

Normal and abnormal labor


In labour which is wrong : which of the following is incorrect :
a-The fetus enters pelvic brim on AP diameter a-continuous CTG with maternal scar
b-Internal rotation for the head b-early cord clamping is a part of active
c- the head exists by extension management
d-Ant shoulder is first c-android pelvis is associated with inc risk of
malformations
Answer : A B or A !!
False about labor :
Regarding mechanism of labour, which is false: a-True labor is suppressed by epidural
a. The rest of the fetal body delivers by flexion anesthesia
b. The commonest fetal position is occipito-anterior b-No cervical changes in false
c. The presenting part enters the pelvic brim in flexion c-the first stage ends with full cervical
d. The external rotation indicates rotation of fetal head to dilation
AP- diameter d- the third stage is the delivery of the
Answer: The external rotation indicates rotation of fetal placenta and membranes
head to AP- diameter e- episiotomy is indicated in all primigravida

A & E !!
The shortest distance between the sacral promontory and Which of the following wrong :
the Answer: android pelvis is common female
lower edge of the symphysis pubis is: pelvis
Normal and abnormal labor
a. Obstetric conjugate
b. Diagonal conjugate
c. True conjugate
d. Interspinous diameter
Answer: Diagonal conjugate
The shortest distance between the sacral promontory and
Term pregnant lady, cervical dilatation
pubic symphysis is:
6 cm, uterine contractions 2 in
A. Biparietal diameter
10 minutes lasting 20 seconds. After two
B. True conjugate
hours, nothing changed. What would you
C. Obstetrical conjugate
do?
D. Diagonal conjugate
a. +give her oxytocin
E. Interspinous diameter
b. immediate C section
Answer: C
c. instrumental delivery
All of the following are true about labor, except:
a. 2nd stage of labor may take 3 hours in a
primigravida with epidural analgesia
All are normally done in labor except:
b. +prolonged 3rd stage is delivery of the placenta after
a. +routine artificial rupture of membrane
20 minutes
when cervix is 4 cm dilated
c. android pelvis
b. asses uterine contractions every 30 min c.
increases the risk of malposition
vaginal exam every 2-4 hours
d. occipito-posterior position is normal in early labor e.
epidural analgesia can be given in active phase of 1st stage
of labor
Regarding management of labour, all the
In a primigravida presented with lower abdominal pain, following are correct except:
one of the following confirms the a. Assessment of vital signs is essential
diagnosis of labour b. Complete blood count
a. Dilated and fully effaced cervix
should be ordered
b. 2 uterine contractions in 10 minutes
c. Fetal assessment by CTG is
c. Passage of small amount of blood per vagina
required
d. The head is at zero station
e. Leakage of liquor d. Oxytocin should be used in all patients
e. Obstetric ultrasound is usually required
Risk factors for uterine rupture in labor
Which of the following is true regarding uterotonic agent
include all of the following except:
administration the active
a. Administration
management of the third stage of labor
of oxytocin b.
a. 5-10 IU oxytocin after delivery of the placenta
Myomectomy
b. 5-10 IU oxytocin after delivery of the baby
c. Previous
c. Oxytocin infusion at crowning till delivery of the
caesarian section
placenta
d. Difficult
d. Ergometrine IV with anterior shoulder delivery
forceps delivery
e. 5-10 IU oxytocin after delivery of the anterior
e. Amniocentesis
shoulder
Answer: E
Regarding mechanism of labor, all of the
During labor, all are true except:
following are true except:
a. The risk of malposition is increased in patients with
a. The presenting part enters the pelvic brim
android pelvis
in a transverse diameter
b. Patients on oxytocin should be monitored with CTG
b. In multipara engagement usually takes
c. The normal blood loss after vaginal delivery is 500-750
place during labor
mL
c. The posterior shoulder delivers first
Normal and abnormal labor
d. The head is engaged when the station is more than d. The fetal head delivers in extension
zero e. The internal rotation indicates rotation of
e. Active management of the third stage will decrease the the fetal head in the AP- diameter
risk for PPH Answer: C
Answer: C
The ischial spine of the mother is the reference point
Which of the following specifically
used to determine which of the following:
suggests obstructed labor:
a. Fetal attitude
a. +severe moulding
b. Presentation
b. fetal distress
c. Fetal engagement d. c. station -2
Lie
d. arrest of dilation
e. Fetal position
e. occipito-posterior position
Answer: C
Regarding false labor, all the following
are correct except
a. The uterine tightness
A 30 year old primigravida is admitted with labor pains. are irregular
On examination her cervix is 2 cm dilated and 1 cm
b. Can be suppressed
long. Name the stage of labor in which she is in
with analgesia
a. Active phase of labour
c. Can start as early as 26
b. Second stage of labour
c. Latent phase of labour weeks’ gestation
d. Late first stage of labour e. d. Are associated with
Not in labour cervical changes
Answer: Latent phase of labor e. The intra-uterine pressure doesn’t exceed
20
Answer: d. Are associated with cervical
changes
All of the following are correct except
a. Growth restricted fetus should be monitored
with continuous CTG
b. Active management of the third stage reduces PPH
c. When only 1/5th of the head is palpable, the head
is considered engaged
d. With active management the normal length of the
3rd stage is 2 hours
e. The risk of malposition increases in patients with
android pelvis
Answer: With active management the normal length of the
3rd stage is 2 hours
Regarding active management of 3rd stage of labor and Regarding labor all of the following are
PPH, only one of the following is correct correct except
a. Syntocinon is contraindicated in cardiac patients a. The length of the first stage of labor in
b. Passage of 600 ml of blood after delivery is considered primigravida is longer than that in
as PPH multigravida
c. A placenta that looks complete rules out retained b. The latent phase is part of the second
placental pieces stage
d. First step includes 5-10 units of syntocinon after delivery c. Second stage of labor indicates delivery of
of the fetus the baby
e. IV syntocinon induces uterine contractions within 5-10 d. The normal duration of second stage
min may reach 3 hours in primigravida
Normal and abnormal labor
e. Oxytoic drugs are commonly used in third
stage of labor
Answer: The latent phase is part of the
second stag

Delivery ( vaginal , assisted & CS )


Not an indication of CS in multiple pregnancy : Day one fever after CS :
a- The first is cephalic and second not a- Atelectasis
b- Monochorionic twins b- Pneumonia
c- Triplet c- Wound infection
A: A A: A
Regarding instrumental vaginal delivery, one of the All fulfill the criteria of normal vaginal delivery
following is true : except:
a. If the head is in -1 station, it is safe to use the forceps a. gestational age 36+4
b. Rupture of membrane is a must to assess the b. bleeding in 3rd stage 300 ml of blood
engagement c. infusion of oxytocin after the delivery of the
c. Forceps can be used to assist vaginal breech delivery anterior shoulder
d. Majority of the cases are elective procedures d. CTG used in monitoring in 1st & 2nd stage
e. Perinatal morbidity is similar to normal delivery of labor
Answer : C Answer: gestational age 36+4 other form 35+6
All are indicators of lower success rates of
About tractional forceps, one is true: VBAC except
a- They are used to deliver the after-coming head A- Obesity
b- They have a sliding lock B- Macrosomia
C- Can be used on intact membrane C- Preterm delivery
D- No cephalic curvature D- Maternal diabetes
Answer: A E- Induction of labor
Answer : C
In a patient who had forceps delivery with complete All the following are delivered vaginally
delivery of the placenta and except
developed sever bleeding with dots, uterus was a. Vertex/non-vertex twins
contracted, what is the most common b. Fetus with an estimated
of bleeding : weight of 4.0 kg
a. Uterine inversion. b. c. Vertex/ Vertex twins
Uterine atony. d. Growth
c. Coagulation defect. restricted fetus
d. Placenta previa. e.Non-
e. Genital tract trauma. Vertex/vertex-
Answer: E twins
About normal vaginal delivery after CS (VBAC), all are true
except:
a. incidence of rupture is 0.5-1% Not an absolute indication of CS ?
b. 70% will deliver vaginally
c. oxytocin use should be avoided - Answer : cord prolapse with a dead fetus
d. +instrumental delivery is contraindicated
e. success rate increases if labor begins spontaneously
Delivery ( vaginal , assisted & CS )
A primigravid patient at 36 weeks of gestation
presents with abdominal pain and vaginal bleeding.
Multiple late decelerations are seen on CTG. The Which of the following statements is true
patient is fully dilated, about traction forceps?
fetus is ROA, +1 station and membranes are ruptured. a. They are used to deliver the after-
What is your next step of coming head
management? b. They have a sliding lock
a. Deliver by immediate Cesarean section c. They have a lower risk of facial injury
b. Instrumental delivery than vacuum
c. Prostaglandins
d. Expectant management

ANC
we do all of the following in ANC except: The EDD for pt whose last menstrual period on 25-
a-Rubella igg 12-2016 :
b-Toxoplasma igg A- 1-10-2017
c-Urine analysis B- 1-9-2017
d-CBC Answer : A
Answer: B
pregnant lady has delivered 35 week child, previous
most accurate method to determine gestational age d27 week, one ectopic
a- certain Last menstrual period pregnancy :
b- Femur length at 25 weeks A- G 3 P2+1
c- transvaginal US at 8 weeks B- G4 p2+1
A:c C- G4 p2
Answer : B

All of the following can be diagnosed by ultrasound at All are serious signs in a 38 weeks pregnant lady
13 week except:- except:
A-Polyhydramnios A. Eye flashes
B-Miscarriage B. Abdominal tightness
C-Ovarian cyst C. vaginal bleeding
D-Multiple pregnancy D. Severe vomiting
E-Anencephaly E. Frontal headache
Answer: A Answer: B
Good antenatal care can prevent or minimize the
G3P2 all of the following can be true except:
following conditions except:
A- she has previous missed miscarriage
a- Hydrops fetalis.
B- she has never had ectopic abortion
b- Iatrogenic prematurity.
C- she might has no alive children
c- Gestational thrombocytopenia.
D- she had a dead child at 28 weeks
d- Folic acid deficiency anemia.
E- she had a still birth
e- pyelonephritis.
Answer: A
Answer C
History about a primi came to ANC for the first time i Pregnant lady at GA 20 wk, all can be a cause for
think the question was large for GA ecexpt :
which of the following is the best : a-GDM
ANC
A-abdominal us at 14 weeks b-Multiple pregnancy
B-abdominal us at 16 weeks c-Wrong date
C-transvaginal us at 8 weeks d-Fibroid
D -B-HCG e-full bladder
ANSWER: C Answer A
At time of EDD the gestational age is :
About EDD, the most accurate :
a-38w
a-Transvaginal us at GA 8 week
b-39w
b-Femural length
c-40w
c-Mother sensation of fetal movement
d-41w
d-LMP
e-42w
A:a
A:c
37 wks pregnant, on examination fundal height was
All of the following can be prevented or minimized by
34, which is incorrect:
good antenatal care except :
A- May be a normal finding
a. Constitutional fetal macrosomia
B- Head circumference is the ideal parameter to
b. Fetal death
detect growth restriction.
e. Breech in labor
C- Around 50% of IUGR cases are missed on
c. Fetal anemia
abdominal exam
Iatrogenic post-maturity
D- oligohydramnios
E- IUFD
A:A
Answer : B
Regarding the accurate estimation of gestational age,
all of the following are true except: All are true about antenatal care except:
a-The ultrasonic estimation at 32 weeks of gestation a. swimming is contraindicated in the 3rd trimester
is usually inaccurate b. exercise results in faster labor
b- In the presence of lactation, ultrasonic estimation c. travel before 36 weeks is allowed for all low risk
is recommended pregnancies
c- It is best done by measuring the crown rump Answer: swimming is contraindicated in the 3rd
length in the second trimester trimester
A or C
Pregnant 30 weeks, has one twin vaginal delivery,
one breach
delivery and one ectopic pregnancy, what is her All of the following can be prevented or
gravidity and minimized by good antenatal care except
parity ? a. Constitutional fetal
macrosomia
A.
b. Fetal death
G5P2+1.
c. Fetal anemia
B.
d. Iatrogenic post-
G4P2+1.
maturity
C.
e. Breech in labor
G5P3+1.
D. G4P2.
E. G3P2+1.
All are routinely done in antenatal care except:
All of the following are true regarding
a. +HbA1c
antenatal care except:
b. platelets count
a. decreases the risk of iatrogenic post maturity
c. screening for asymptomatic
b. decreases complication on mother and fetus
bacteriuria
in mother with medical illness
d. pelvic ultrasound
c. prevent iron deficiency anemia
e. urinalysis
ANC
d. +prevents fetal anomalies

Regarding ANC , all are true expect ?


a. +swimming is contraindicated in the 3rd
All of the following statements are correct about
trimester
dating pregnancies except
b. exercise results in faster labor
Answer: It is dated from the last day of the
c. travel before 36 weeks is allowed for all
last menstrual period
low risk pregnancies

A primigravid female presents to the office for her


A patient 35 weeks pregnant presents to the clinic first prenatal visit. She is 12 weeks pregnant. All of
for a followup visit. On US her fetus is in a breech the following investigations are ordered except
position. Which of the following is the next best a. HbsAg
step of management? b. Rubella IgG
a. Schedule a repeat visit in 2 c. Urine culture
weeks d. Glucose tolerance test
b. External cephalic version Answer: Glucose tolerance test
c. Deliver by elective cesarean section

All the following can be diagnosed by


ultrasound examination at 10 weeks
A patient in the first trimester is found to have a gestation except
positive urine culture for 100,000 colony forming a.Chronicit
units. Which of the following is the best line of y b. Twins
management?
c. Corpus luteal
a. Admission and IV antibiotics
cyst
b. Oral antibiotics for 5-7 days
d. Missed
c. Single dose of oral antibiotics
miscarriage
e. Placenta
previa
Regarding the accurate estimation of gestational age,
all of the following are true
except: Good antenatal care can prevent or minimize all
a. The ultrasonic estimation at 32 weeks of of the following conditions, except:
gestation is usually inaccurate a. Placenta previa
b. In the presence of lactation, ultrasonic b. Iron deficiency
estimation is recommended anemia c. RH
c. It is best done by measuring the crown rump immune reaction d.
length in the second trimester Pyelonephritis
d. It is calculated from the first day of the last e. Fetal death due to placental insufficiency
menstrual period Answer: A
e. It reduces the risk of post-maturity
Answer: A?

Which of the following statements is correct


Which of the following statements about dating
a. Tetanus vaccine is safe in the third
in pregnancy is incorrect
trimester
Answer: The date is calculated from the last day
b. Rubella vaccine is safe in the first of the last menstrual period
trimester
Answer: Tetanus vaccine is safe in the third trimester
ANC

All the following are routine tests in an antenatal All of the following are true about the fetal skull
care visit except except
a. OGTT a. The posterior fontanel is diamond in shape
b. Antibody b. During labor severe molding can be an
screen indicator of difficult labor
c. CBC
d. Rubella IgG
If a patient’s last menstrual period was 25/12/2015, The earliest time fetal movement can be detected
her expected date of delivery (EDD) will be by the mother is
a. a. 10
1/10/2016 weeks
b. b. 16
5/10/2016 weeks
c. c. 24
1/9/2016 weeks
d. d. 6
5/9/2016 weeks
All can be detected on 1st trimester All can be diagnosed by ultrasound in the
ultrasound except: 3rd trimester, except:
a. microcephaly a. renal
b. gestational age agenesis
c. ensure intrauterine pregnancy b. Turner
d. chromosomal anomalies c. facial cleft
e. number of fetuses d. oligohydramnios

Pregnant lady at 37 weeks gestation with


symphysis fundal height of 34 cm, all of the
The following are causes for large for date
uterus, except: following are possible causes
except
a. Multiple
a. Uncertain dates
pregnancy
b.
b. Transverse lie
Oligohydramni
c. Full bladder
os
d.
c. Transverse
Polyhydromnios
lie
e. Wrong date
d. Estimated fetal weight of 4000 grams
e. PROM
All are causes for large-for-date uterus, except:
a. bilateral renal agenesis
b.anencepha
ly
c. wrong
date
d. esophageal atresia
e. full bladder
Fetal abnormalities, growth and monitoring
all causes asymmetrical IUFR except : biophysical profile contain all these except :
A- chromosomal abnormalities a-fetal movement
B- SLE b-fetal breathing movement
C- Thrombophilia c-Doppler
D- renal failure A:c
Answer : A

Regarding accurate estimation of


gestational age, all the following are correct
except:
a- is best done by measuring CRL in the first
All following cause asymmetrical IUGR except :
trimester.
a-SLE
b- use of COCP may associated with wrong
b-HTN
estimation.
c-RF
c- BPD is best done at 12 weeks gestation.
d-chromosomal abnormalities
d- ultrasonic estimation at 26 weeks
Answer D
gestation is usually inaccurate.
e- reduce the risk of unnecessary preterm
birth.
Answer: C

concerning small for gestational age fetus,


all the following are correct except:
All of the following matches except :
a- may be concequence to maternal
a. banana sign in neural tube defect
cigarette smoking.
b. anencephaly and polyhydramnios
b- should be delivered before 37 weeks
c. esophageal atresia and oligohydrominos
gestation
d. nephropathy obstruction and pulmonary hypoplesia
c- may be associated fetal anomalies.
Answer: C
d- best diagnosed by ultrasound.
e- approximately half of the cases are
missed on routine palpation.
Answer: B
regarding fetal non-stress the change in the fetal heart
in partogram all of the following are
associated with fetal
included except :
:movement is
a- cervical dilation
.a- increased beat to beat variability
b- patient profile
.b- transient accelerations
c- past medical or surgical history
.c- prolonged tachycardia
d- head staging
.d- early decelerations
e - uterine contractions
.e- variable decelerations
A:c
Answer is B
Question about not part of partogram :
which one of the following causes an increases in AFP?? The answer is previous hx of neonatal
answer : myelomeningeocele admission
Fetal abnormalities, growth and monitoring
One of the following does not explain
All of the following cause increased in alpha feto protein :
reduced variability in fetal heart tracing:
A. Omphalocele
a. Barbiturates ingestion
B. Turner
b. Sleeping phase
C. Trisomy 18
c. Fetal stimulation
D. Trisomy 21
d. Asphyxia
E. Meningocele
e. Prematurity
Answer: D
Which of the following is NOT part of partogram:
Whats abnormal regarding CTG :
A- fetal position
Thr answer is : 4 late decelartions
B- fetal descent
C- maternal contraction in 30 min
D- fetal heart rate
E- liquor status
Regarding growth restriction, only one of the following is The following are biophysical profile
true: components, except:
a. May be caused by dieting in pregnancy a. Fetal weight
b. Usually occurs after 35 weeks of gestation b. Fetal heart pattern
c. Could be a presentation of chromosomal abnormalities c. Fetal tone
d. Usually associated with polyhydromnios d. Liquor volume
e. Head circumference is the ideal parameter to detect e. Fetal breathing movement
growth restrictions Answer: A
Regarding causes of fetal growth restriction, only one of
The following factors affect normal fetal
the following is correct
weight except:
a. Race
a. Parity
b. Hot climate
b. Race
c. Dieting in pregnancy
c. Maternal built
d. Gestational diabetes
d. Climate and weather
e. Anemia
e. Fetal sex
Answer: D
Answer: e. Anemia
Of the following, only one is correct:
a. umbilical artery doppler is of no value
after 34 weeks gestation
b. meconium in the trachea is a very
About growth restriction, which one of the following rare finding
statements is correct c. +oligohydramnios is found in all IUGR
babies
Answer: most SGA babies have normal growth pattern d. small for gestation age babies are all
growth restricted
e. IUGR is defined as weight below 15th
percentile

All are causes for decreased variability on CTG, except: Which of the following is false about the
a. opioids CTG in a term baby
b. prematurity a. Normal variability is >5 bpm
c. sleeping b. Baseline heart rate is 100-160 bpm
Fetal abnormalities, growth and monitoring
d. +epidural c. It can be done more frequently in
analgesia e. higher risk patients d..
hypoxia Answer: Baseline heart rate is 100-160 bpm
Regarding cardiotocograph (CTG) at
term all the following are normal
findings except
a. Baseline heart rate of 145 bpm
All the following are shown in partogram in labour except b. 8 uterine contractions in 10 minutes
a. Number of uterine contractions in 30 minutes c. One shallow early deceleration in the
b. Cervical dilatation second stage of labor
c. Liquor status d. Two accelerations
d. Maternal pulse in 30 minutes
e. Urine dipstick for protein e. Beat to beat
variability of 15 bpm
Answer: 8 uterine contractions in 10
minutes
Regarding partogram in labor, all the following are
correct except All are components of partogram
a. It is essential to avoid unnecessary caesarean delivery except
b. It can demonstrate arrest of labor : a. maternal pulse
c. It is done in second stage of labor b. fetal presentation
d. It is the best way to assess progress of labor c. cervical position
e. It can demonstrate uterine hyperstimulation d. contractions

All of the following are true about a


patient with androgen insensitivity
syndrome
Which of the following statements is incorrect about
except
congenital anomalies
a. The patient will have normal
a. Tracheo-esophageal fistula is associated with
external male genitalia
oligohydramnios
b. Patient will have a karyotype with
b. Neural tube defects are associated with banana sign
XY
c. Anencephaly is associated with polyhydramnios
c. It is an androgen
Answer: Tracheo-esophageal fistula is associated with receptor mutation
oligohydramnios
Answer: The patient will have normal
external male

About congenital anomalies, all are true


except:
a. overall incidence of major congenital
All are causes for increased AFP except:
anomalies is 3%
a. hydatiform mole
b. +most common cause is chromosomal
b. b. anencephaly
abnormalities
c. +Down syndrome c. common presentation is 1st
d. fetal demise
trimester miscarriage
e. multiple pregnancy
d. may not be evident at birth
e. might be caused by environmental
factors
Fetal abnormalities, growth and monitoring
Regarding congenital anomalies : limb defects, chorioretinitis,
cerebral cortical atrophy, cutenous lesion :

A. Rubella
B. Varicella
C. Cmv
D. toxoplasmosis
Urinary incontinence & genital prolapse
Regarding stress incontinence, all are true except : All of the following associated with stress
a. Involuntary leakage of urine occurs with sneezing, incontinence except:-
coughing and laughing A. Treated by exercise and surgery
b. Is the most common type of incontinence B. Cystometry used in diagnosis
c. Is best treated with anticholinergic medications C. Avoid caffeine
d. During cytometer, there are no uninhibited detrusor D. Anticholinergic drugs the most
contractions during filling commonly used medication
Answer : C Answer: D
About stress incontinence, all are true except:
Regarding urodynamic study one is true:
a. can be associated with urge incontinence
A- flow rate more than 15 ml/ sec
b. can be treated effectively by anti-cholinergic
B- most common cause of overflow
medications
incontinence in female in urethral
c. they have urine leak upon coughing or sneezing
outlet
d. vaginal childbirth is a risk factor
obstruction
Answer: B
C. bladder volume 200-300 cc
In another form:
Answer A
E. Alpha receptors present in the body of bladder
About urge incontinence :
a-Involuntary loss of urine preceded by ugre to void Which of the following is NOT a treatment
b-Caused by uninhibited bladder detrusor contraction of urge incontinence :
Answer A Answer : anterior vaginal colporrhaphy

Urge incontinence all true except:


urge incontinence one is true :
Answer: tension free vaginal tape (TVT)
Answer : incontinence follow strong desire to void
is one of the operative option
The following ligaments that provide the most support to the All of the following are risk factors for
uterus are the : genital prolapse except
a. Broad ligaments a. White race
b. Round ligaments b. Exercise
c. Arcuate ligaments c. Menopause
e. Cardinal ligaments d. Hysterectomy
A:E Answer: Exercise
All are true about urodynamic
studies except:
a. +bladder capacity is 200-300 ml
b. first desire to void occurs
Regarding genital prolapse, Which of following is NOT true?
at 150-250 ml c. flow rate
during voiding is > 15
- Answer: common in nulliparous
ml/sec
d. it's important for accurate diagnosis of
urge incontinence
e. residual volume should be less than 50
ml
All of the following may be
All of the following are common symptoms of urinary
considered in the management of
incontinence except
urge incontinence,
a. Stress incontinence
except:
Urinary incontinence & genital prolapse
b. Overflow incontinence c. a. Intravesical therapy (e.g.
Urge incontinence Botulinum toxin)
d. Frequency b. Tolterodine and oxybutynin
e. Straining to void c. Bladder training
Answer: Straining to void d. Retro-pubic bladder neck
suspension operations
e. Avoid caffeine and based drinks
Answer: D
One of the following describes genuine stress
incontinence About genital prolapse, one is wrong:
a. Incontinence that occurs during sleep a. 2nd degree uterine prolapse is descent
b. Incontinence due to involuntary bladder detrusor of the cervix in the introitus
contractions b. vaginal hysterectomy is a suitable
c. Involuntary urine loss preceded by a strong desire to void management
d. Incontinence that occurs after coughing or sneezing c. +uretrocele is descent of the
e. All involuntary urine loss vagina through the upper 1/3 of
Answer: d. Incontinence that occurs after coughing or posterior wall
sneezing
Genital organs are supported by all of the following Factors that increase the risk of genital
except: prolapse include all of the following
except:
a. Transverse cervical ligaments
a. Forceps delivery.
b. Pelvic fascia
b. Hysterectomy.
c. Levator ani muscle
c. Cesarean section.
d. Round ligament d. Caucasian race.
e. Uterosacral ligament e. Chronic cough.
Answer: D Answer: C
All of the following are causes of
All of the following are true about genital prolapse except
congestive dysmenorrhea except
a. It is found in 25% of multiparous females
a. PID
b. It is the main cause of stress incontinence
b. Asherman’s syndrome
c. Procidentia is prolapse of the uterus through the introitus c. Pelvic adhesions
d. More common in Asians than Blacks
d. Pelvic congestion
Answer: It is the main cause of stress incontinence
e. Genital prolapse
Fibroid
All of the following are risk factors for uterine fibroids All of following risk factor for fibroid except :
except : A. COCP
a. Nulliparity B. black race
b. White race C. nulliparity
c. PCOS D. family history
d. Family history E. increased estrogen level
e. Obesity Answer: A
The most common tumor in females of
All fowllowing are risk for fibroid except :
reproductive age is:
a-Reterograde menses
a. Sarcoma
b-African
b. Leiomyoma
c-Nullparity
c. Adenocarcinoma
d-Family history
d. Adenomyosis
Answer : A
A:B
Uterine fibroid can cause all of the
Regarding fibroids all are true except: following in pregnancy except:
A. Red degeneration is common in pregnancy a. IUGR
B. HRT could be used to treat postmenopausal b. +fetal anomalies
females with fibroids c. recurrent miscarriage
C. Most commonly present with menorrhagia
d. abnormal presentation
Answer: B
e. delivery by C section
Regarding fibroid presentation, all of the
All of the following are true about uterine fibroids following are known symptoms except:
except a. Menorrhagia
a. Red degeneration most commonly in b. Intermenstrual
pregnancy bleeding
b. Mostly present with menorrhagia c. Sub-fertility
c. Submucosal fibroid can be treated through d. Asymptomatic
hysteroscopic removal e. Urinary frequency
Answer: B
All of the following are recognized risk
All of the following are risk factors for uterine factors for fibroids except:
fibroids except a. Family history
a. Multiparity b. History of uterine
b. Black race surgery
c. Smoking c. Black race
d. Family history d. Null parity
Answer : C e. High estrogen levels
Answer: B
30 years female, para 2, presented with menorrhagia
and found to have intramural
fibroid, best management:
a. mirena
b. +myomectomy
c. GnRH analogue
d. COCPs
e. hysterectomy
Endometriosis, Adenomyosis & Endometrial CA
The definite diagnosis of endometriosis is made
The prognosis of endometrial carcinoma is worse
by:
with all except:
a-hysteroscopy
A-Elderly
b-laparoscopy
B-hypertention and DM
c-abdominal ultrasound
Answer: B
A:B
all are risk of endometrial carcimona except:
a- Obesity
b- Lynch syndrome which is wrong about endometrial cancer:
c- Endometrial polyp A- 25% affect premenopausal women
d- turner B- mostly found in stage 1

Regarding endometrial only one of the following is


correct:
What's the risk of complex atypical endometrial a- clinical examination is usually very suggestive of
hyperplasia to progress into the diagnosis.
endometrial Ca: b- radiotherapy is very effective primary therapy.
a. 1% c- lymph node involvement is related to the grade of
b. 5% the tumor and the depth of
c. 15% the myometrial invasion.
d. 25% d- Type 2 carcinoma is HPV related.
Answer: 25% e- serous papillary type carries a better prognosis
than endometroid type.
Answer: C
All of the following statements are true about
Adenomyosis except
Endmoetrial CA:
a. It is diagnosed by laparoscopy
Answer : Most commonly adenocarcinoma
b. Hysteroscopy can show uterine adhesions
c. It is a recognized cause of recurrent miscarriages
Symptoms of endometriosis include all of the
About endometriosis, one is following except:
false: a.
a. has a genetic predisposition Dysmenorrhea
b. +endometrioma can't increase more
b. Dyschezia
than 5 cm in diameter
c. Chronic pelvic
c. ovarian injury during surgery
can attribute to infertility pain
d. transformation to malignancy is rare d. Vaginal
e. 1/3 of cases are diagnosed through infertility discharge
investigations e. Dyspareunia
Answer: D
In a 20 year old single complaining of All of the following are used in the treatment
severe dysmenorrhea because of of endometriosis,
endometriosis, the treatment of except:
choice is: a. NSAIDs
a. Continuous oral progesterone b. COCPs
b. Mirena c. Oral progesterone
c. Cyclical d. Conjugated
COCP estrogen
d. GnRH e. GnRH analogue
analogue Answer: D
Endometriosis, Adenomyosis & Endometrial CA
e. Bilateral oophorectomy
Answer: A

A 30 year old patient presents with


All of the following are associated with
infertility, dyspareunia and dysmenorrhea
endometriosis except :
for a number of years. What is the next best
a. Early pregnancy
investigation for this patient?
loss
a. HSG
b. Ectopic pregnancy
b. Laparoscopy
c. Late menopause
Answer: Late menopause
Regarding endometriosis all the following are
correct except
a. Progressive disease The most common presentation of
endometriosis is
b. Is best diagnosed by pelvic
a. Dyspareunia
ultrasound
b. Infertility
c. Improves during pregnancy c. Menorrhagia
d. Is benign d. Chronic pelvic pain
e. Usually no abnormal findings
on physical examination
35 years old P3 has endometriosis and doesn't
wish to preserve fertility, best
All of the following increase the risk of endometrial
management:
cancer except:
a. continuous OCPs for 6-9 months
a. +progesterone therapy
b. hysterectomy and bilateral salpingo-
b. obesity
oophorectomy
c. tamoxifen
c. danazol
d. granulosa theca cell tumor
d. GnRH
e. progesterone secreting IUD
About endometrial cancer, only one is correct: All are true about endometrial cancer except
a. progesterone therapy is used for a 59 years a. obesity is a risk factor
old patient with atypical b. serous type is associated with good prognosis
hyperplasia c. baseline treatment for all patients is
b. tamoxifen has a recognized hysterectomy and bilateral
association with endometrial salpingooophorectomy
hyperplasia d. post-menopause women have worse prognosis
than pre-menopause
Regarding endometroid endometrial carcinoma,
one of the following statements is Which one of the following has the highest rate
true: of progression to uterine cancer?
a. Estrogen dependent a. Uterine leiomyoma.
b. Occurs mainly before the b. Simple hyperplasia with
atypia.
age of 50 c. Usually presents
c. Complex hyperplasia.
with pelvic pain
d. Complex hyperplasia with
d. Usually advanced stage at the time of diagnosis atypia.
e. The diagnosis is confirmed by increased e. Simple hyperplasia.
endometrial thickness on ultrasound Answer: D
Answer: A
All of the following are risk factor for
Which of the following is true about endometrial
endometrial carcinoma except:
cancer
a. Obesity.
Endometriosis, Adenomyosis & Endometrial CA
b. Unopposed estrogen a. Lymph node invasion is related to depth of
therapy. myometrial invasion and
c. Late menopause. grade of the tumor
d. Tamoxifen b. Type II endometrial cancer is not estrogen
therapy. related (or dependent*) but HPV related
e. Multiparity
Answer: E
The prognosis of endometrial carcinoma is
worse with all the following except
a. Myometrial invasion
b. When the tumor is poorly The most important determinant for cancer
differentiated in endometrial hyperplasia is
c. In elderly a. Cellular atypia
d. When there is cervical b. Grade
involvement
e. Endemetroid type
Answer: e. Endemetroid type
Risk factors for endometrial carcinoma include all
of the following except
a. Late
menopause
b. Obesity
c. COC
d. Nulliparity
e. Family history

Cervical disease & CA


Regarding cervical ectropion, all of the following
All the following statements are true regarding
are true except :
screening for cervical cancer
a. It is commonly seen in post-menopausal
except:
women.
Cervical disease & CA
b. Cervical smear is mandatory before a- normal squamous epithelial cells give a mahogany
treatment. brown stain after
c. Post coital bleeding is a common applying lugol's iodine solution.
presentation. b- leukoplakia in colposcopy is a normal finding and
d. Can be caused by combines should be ignored.
contraceptive pills. c- screening starts at age 21.
Answer A d- acetic acid is used in colposcopy.
e- women with smear showing HISL should be
referred to colposcopy.
Answer: B
Regarding cervical ectropion, all the following are
correct except
Regarding HPV all true except:
a. It’s a cause of post coital bleeding
A- HPV is double stranded DNA
b. Is pathological in post-menopausal women
B- HPV vaccine is contraindicated during lactation
c. Is associated with increased vaginal discharge
C- HPV 16 and 18 account for 70% of cases
d. Is more common during pregnancy
D- Most of HPV infections resolve after 12 months
e. Is increased in pill-users
Answer: B
Answer : E

Which of the following is correct:


The infectious agent that is involved in the
A-Cervical ectropion is a benign condition and
development of cervical cancer is :
requires no managemen
a. Herpes b virus .
B- Adenomyosis is diagnosed by endometrial
B.Human papilloma virus
biopsy
c . Syphilis
C- Adenomyosis is more in nullparous women
d. Gonorrhea
Answer: A (mostly)
Answer : B
Regarding cervix cancer , all of the following are
true except :
a.Multiparous women have an increase risk .
b.Smoking is a co-factor in the etiology of the
Which is wrong about cervical cancer : disease.
Answer : Adenocarcinoma is the most common c. Pap smear is only a screening test for the
type disease.
d. Squamous carcinoma is the most common
cell type.
e. Rarely occur before the age of 40.
Answer: B
All of the following are true about HGSIL Which of the following statements is true about CIN
except (cervical intraepithelial neoplasia)?
a. Cone biopsy is indicated if the cervix if
colposcopy is inadequate a. Persistent CIN I can be treated with LEEP
b. Cone biopsy is indicated can be b. Patients who underwent cone biopsy don’t
therapeutic require followup
c. Endometrial sampling is always c. CIN is a cytological diagnosis
needed d. CIN III is defined as dysplasia involving the
Answer: Endometrial sampling is always needed whole epithelium with invasion into stroma
Which of the following statements is incorrect Regarding cervical ectropion, all the following are
about cervical cancer correct except
Answer: in Stage IA, survival is 60% a. It’s a cause of post coital bleeding
Cervical disease & CA
b. Is pathological in post-menopausal women
c. Is associated with increased vaginal
discharge d. Is more common during
pregnancy
e. Is increased in pill-users
Not a cause of post-coital
bleeding: a. cervical polyp About cervical cancer, all are true except:
b. cervical carcinoma a. +cervical smear is diagnostic
c. +endometrial polyp b. with stage Ia, 5 year survival is 95%
d. cervical ectropion c. squamous cell carcinoma is the most common
e. cervicitis
In high grade intraepithelial lesions, all are
true except:
a. +endocervical curating and endometrial
About CIN:
biopsy are indicated
a. it is a cytological description
b. cone biopsy is indicated if the squamocolumnar
b. hysterectomy is a standard treatment of CIN III
junction couldn't be visualized
c. CIN I can regress in 60% of case
c. mostly caused by persistent infection with
HPV 16, 18
d. can be asymptomatic

Ovarian mass
About ovarian tumors all true except :
A 75 year old Japanese lady complaining of
A. Granulosa cell tumor cause endometrial
hematemesis was found to have bilateral ovarian
hyperplasia
masses. She most likely has:
B. Endometeioma cause high ca 125
A- Krukenberg tumor
C. Ca 15.3 is high in yolk sac tumor
B-Fibroma
D. Germ cell tumors are the most common
C-Granulosa cell tumor
ovarian cancers in children and adolescents.
D-Teratoma
E. Molar pregnancy are associated with theca
E-Clear cell carcinoma
luteal cyst.
Answer: A
Answer: C
Granulosa cell tumor all true except:
Regarding epithelial ovarian cancer, one is
A- best treated with surgical rather than medical
correct:
B- Associated with endometrial hyperplasia
a. COCP is a risk factor
C- Cause precious puberty in young girls
b. Carries good prognosis if diagnosed early
D- It secrete progesterone
c. Staging of the disease is based on radiological
E- It is a germ cell tumor incidence is more in children
findings and Ca125 level
and adolescent
d. Familial in 50% of cases
Answer D (granulosa cell tumor secrete estrogen)
Ovarian mass
e. Borderline type is best treated by
chemotherapy
Answer : B
A patient presents with a solid unilateral ovarian mass
on ultrasound and has a All of the following are the features of
significantly elevated LDH level, negative AFP and HCG. ultrasound in ovarian cancer except:
The most likely type of her A. Solid mass
ovarian mass is a(n) B. septation > 3 mm
a. Immature teratoma C. Size more than 10 cm
b. Choriocarcinoma D. Ascites
c. Endodermal sinus tumor E. Pelvic mass
d. Dysgermninoma Answer: C
Answer: Dysgerminoma
Most common ovarian neoplasm in pregnancy is: case about ovarian cancer , AFP is elevated, HCG
a- corpus Luteal cyst and LDH is normal … most likely diagnosis :
b-granulosa cell tumor a- Choriocarcinoma
c-endometriod b- Dysgerminoma
d-clear cell c- Endodermal sinus tumor
A:A Answer : C
one is epithelial ovarian tumor:
a- Granulose cell tumor
young women presented with SOB, found to
b- Clear cell tumor
have pleural effusion and bilateral
c- Teratoma
ovarian masses, what is your diagnosis?
d- Endodermal sinus tumor
Answer: Fibroma
e- fibroma
Answer: b
which one is the epithelial type of ovarian cancer :
answer is: clear cell carcinoma
Regarding epithelial ovarian cancers, one
is true:
a. The majority of epithelial cancers are
familial
One of the following ovarian tumors could be b. CA 125 is not helpful in the follow up of
bilateral in 10-15% of cases: ovarian cancer
Answer: dysgerminoma c. Endometroid type 30% coexist with primary
endometrial cancer
d. Usually present with acute symptoms
e. Borderline type carries worse prognosis
than invasive type
Answer: C

One of the following is a malignant germ cell tumor of Meigs syndrome is associated with the
the ovary: following ovarian neoplasm:
a. Thecoma a. Brenner tumor
b. Androblastoma b. Non- gestational
choriocarcinoma
c. Mucinous carcinoma
c. Yolk sac tumor
d. Dysgerminoma
d. Serous
e. Mature cyst teratoma adenoma
Answer: D
e. Fibroma
Ovarian mass
Which of the following is true about epithelial
ovarian cancer
a. If diagnosed in early stages it has a good
prognosis
b. Serous has a better prognosis than
endometroid
c. Oral contraceptives are not a protective factor
A 21 year old female patient presents with a solid
unilateral ovarian mass on ultrasound and has a Ovarian neoplasms most commonly arise from
significantly elevated alpha a. Metastatic disease
feto-protein level. The most likely type of her b. Ovarian germ cells
ovarian mass is a(n) c. Ovarian epithelium
a. Immature teratoma d. Ovarian sex cords
b. Choriocarcinoma e. Ovarian stroma
c. Endodermal sinus tumor
Answer: Endodermal sinus tumor
Only one of the following tumors produce
Regarding ovarian tumors, all of the following are
estrogen
germ cell tumors except
a. Thecoma
a. Yolk sac tumor
b. Endodermal
b. Teratoma
sinus tumor
c. Dysgerminoma d. c. Choriocarcinoma
Choriocarcinoma e. d. Dysgerminoma
Endometroid tumor
e. Serous cyst adenoma
Answer: Endometroid tumor
Answer: Thecoma

One of the following ovarian tumors is most likely to


result in virilisation of a 35 years old woman
a. Sertoli-Leydig cell tumor
b. Thecoma Inhibin is a tumor marker
c. Brenner tumor in:
a. +granulosa cell tumor
d. Mucinous
b. dysgerminoma
cystadenoma
e. Dysgerminoma
Answer: Sertoli-Leydig cell tumor

About ovarian cancer, one is false:


a. epithelial is the most common type
b. +tubal ligation
increases its risk
c. endometrioid type may be associated with
endometrial carcinoma
d. endometriosis is associated with clear cell
type
e. multiparity reduces its risk
Vaginal discharge
Regarding bacterial vaginosis, one of the
following is true:
All of the following can be sexually transmitted, except:
a. It is a rare vaginal infection
a. Trichomonas vaginalis
b. It is treated with anti-fungal agents
b. Chlamydia
c. It is associated with fishy smell discharge
c. Vaginal candidiasis
d. It is always symptomatic
d. HIV
e.It is usually associated with severe vulvar
e. Gonorrhea
itching
Answer C
Answer: It is associated with fishy smell
discharge

About chlamydia trachomatis, all true except :


About chlamydia trachomatis, one is wrong:
A. Its gram negative intracellular bacteria
a. gram negative intracellular organism
B . Cause mucopurulent endocervical
b. may be a cause for neonatal pneumonia
discharge
c. most common cause of vaginal discharge
C. Sexually transmitted
d. diagnosed by endocervical swab
D. Cause severe symptoms
e. treated by erythromycin during pregnancy
E cause PID and ectopic pregnancy.
Answer: most common cause of vaginal discharge
Answer: D

Regarding bacterial vaginosis one is false


One of the following is NOT correct about candidal :
vulvovaginitis: A- no need to treat the partner
a. Intense itching B- metronidazole is the DOC
b. Common in pregnancy C- Whiff test +ve
c. pH > 6 D- most common cause is lactobacilli
Answer: pH > 6 ANSWER : D

About candidiasis, all are true except:


Which of the following is False about vaginal candidiasis?
a. +recurrence is not common
b. fluconazole is not safe during pregnancy
- Answer:usually asymptotic,it’s usually itchy with white
c. vulva will be edematous and inflamed
discharge
d. whiff test is negative
e. occurs more during pregnancy

Regarding chlamydia trachomatis infection,


all of the following are true except:
All of the following can be sexually transmitted, except:
a. Is a common cause of vaginal discharge
a. Trichomonas vaginalis
b. May predispose to perihepatic adhesions
b. Chlamydia
c. May be treated with erythromycin or
c. Vaginal candidiasis
tetracycline
d. HIV
d. Is a gram negative intracellular organism
e. Gonorrhea
e. Can cause neonatal pneumonia
Answer: C

A young female patient presents with foul smelling Regarding trichomonas vaginalis, all are true
discharge and abdominal pain. She except:
Vaginal discharge
is diagnosed as a case of acute PID. All of the following a. Presents with green vaginal discharge
are possible steps in the b. Complications during pregnancy
management except include low birth weight and
a. Oral clindamycin prematurity
b. Oral doxycycline c. KOH wet mount shows motile
c. Hysteroscopy with D and C flagellated microorganisms
d. Laparoscopy d. Infection during pregnancy can
Answer: Hysteroscopy with D and C cause neonatal pneumonia
e. Male partner should be treated

Regarding vulvovaginal candidiasis, all of the


following are true except:
All of the following are true about Trichomonas
a. Less common in combined oral
vaginalis vaginitis except
contraceptive pill users
a. Yellow frothy discharge is often observed
b. The discharge is thick, cottage cheese like
b. Treated with metronidazole
c. It causes erythema with
c. Sexually transmitted
excoriation
d. Wet mount shows flagellated motile micro-
d. Main symptom is
organisms
pruritus
e.
e. KOH wet mount shows budding
Answer: e
yeast cells and pseudohyphae
Answer: A
All of the following are true about candidal All of the following are true about
vulvovaginitis except Chlamydia trachomatis except
a. It is a gram negative
a 80% of the cases are caused by candida
intracellular bacteria
albicans
b. It causes severe symptoms
b. Presence of a copper IUCD is a risk factor
c. It can lead to ectopic pregnancy
c. It is the most common cause of vaginal discharge in a and infertility
pregnant woman Answer: It causes severe symptoms
Regarding candidal vaginitis, all the
following are correct except
All the following can be transmitted sexually except a. Is treated with doxycycline
a. Gonorrhea b. Is associated with thick
b. Trichomonas vaginalis white discharge
c. HIV c. May be associated with
d. Vaginal candidiasis dyspareunia
e. Chlamydia d. Frequently associated with intense
Answer: Vaginal candidiasis vulval pruritus
e. Whiff test is negative
Answer: a. Is treated with doxycycline
Which one of the following is true about
Vulvo-vaginal candidiasis occurs more frequently in the bartholin’s abscess
following except: a. It is usually bilateral
a. Thyrotoxicosis b. It presents as a painful swelling to one side
b. Pregnant of the clitoris
c. Diabetes c. It is usually asymptomatic
d. On oral contraceptive pills d. It is treated by marsipulization
e. On Long term antibiotic therapy Answer: b. It presents as a painful swelling to
one side of the clitoris*
Vaginal discharge
About bacterial vaginosis, one is
wrong:
a. pH is elevated
Which of the following is true about bacterial vaginosis b. profuse foul smelling
Answer: It causes fishy smell and watery discharge thick vaginal discharge
c. may be a risk for
postpartum endometritis
d. no pruritus
About chlamydia trachomatis, one is wrong: Not a cause of superficial
a. gram negative intracellular organism dyspareunia:
b. may be a cause for neonatal pneumonia a. lactation
c. +most common cause of vaginal discharge b. +pelvic adhesions
d. diagnosed by endocervical swab c. vulvar and vaginal
e. treated by erythromycin during pregnancy candidiasis
A d. psychological
e. vaginal atrophy
Female pregnant in the first trimester. Chlamydia
test was positive. The best treatment option is:

A. Erythromycin
B. Azithromycin
C. Doxycycline
D. Metronidazole
Not risk factor for candida:
A. Thyrotoxicosis
B. COCP
About bacterial vaginosis, one is wrong: C. Pregnancy
a. +sexually transmitted D. DM
b. clue cells on wet mount are diagnostic
c. usually asymptomatic candidal vaginitis, all the following are correct
d. often presents with foul smelly discharge except
e. pH is more than 4.5 Answer : Is treated with doxycycline

Vulvo-vaginal candidiasis occurs more frequently


in the following except:
A. Thyrotoxicosis
B. Pregnant
C. Diabetes
D. On oral contraceptive pills
E. On Long term antibiotic therapy
Menorrhagia
we use all of the following in menorrhagia except: Dysfunctional uterine bleeding (DUB), which of
a-Danazol the following is false :
b-norethisterone a- It can be ovulatory
c-Gnrh analogue b- It is common in perimenarchal age
d-Tamoxifen c-it could be due to endometrial polyp
e-Mefanamic acid A:c
A: d
Investigations for a 30 years old female P4+0,
Regarding dysfunctional uterine bleeding (DUB), which complaining of menorrhagia, include all of the
of the following is false: following except:
a. It is commonly managed with hormonal therapy a. Hysteroscopy and dilatation and
b. It can be due to large submucosal fibroid curettage
c. c. It is common in perimenarchal age b. Thyroid function tests
d. d. It can be ovulatory c. Cervical smear
e. e. It can be treated with hysterectomy d. Complete blood count
Answer: It can be due to large submucosal e. Vaginal ultrasound
fibroid Answer: Hysteroscopy and dilatation
and curettage

About management of menorrhagia, all are true except:


A- NSAIDs decrease bleeding by 25-50%.
B -Tranexamic acid should not be given to pt with
Regarding dysfunctional uterine bleeding,
bleeding disorder
which of the following statements is
C- Progestin is used for anovulatory DUB
incorrect?
D- COCP is contraindicated to be used to manage
a. Can be ovulatory
menorrhagia in 45 years female
b. Occurs only in menopausal
E- Endometrial ablation is inducing iatrogenic asherman
females
syndrome
Answer: Occurs only in menopausal females
Answer: B

Regarding menorrhagia, all of the following


statements are true except:
a. Adenomyosis is a
All are recognized causes of organic menorrhagia recognized cause
except: b. Don’t occur in ovulatory
a. endometriosis cycle.
b. granulosa cell tumors c. Could be due to ineffective contraction of
c. fibroids myometrium vessels.
d. +Sheehan syndrome d. Can be treated by endometrial ablasion.
e. Is defined as prolonged and regular
menstrual bleeding.
Answer: C
A 41 year old female who is a smoker,
A young patient complains of DUB. All of the presents complaining of DUB. All of the
following can be used in her management except following
Answer: Copper IUCD can be lines of management except
Answer: Combined oral contraceptives
Menorrhagia
Investigations for a 30 years old female
Congestive (secondary) dysmenorrhea can be caused P4+0, complaining of menorrhagia,
by all of the following except include all of the following except:
a. Pelvic adhesions a. Hysteroscopy and dilatation
b. Cervical stenosis and curettage
c. Pelvic inflammatory b. Thyroid function tests
disease d. Adenomyosis c. Cervical smear
e. Asherman’s syndrome d. Complete blood count
e. Vaginal ultrasound
All the following modalities can be used to treat DUB
(Dysfunctional uterine bleeding) except Normal duration of menstrual
a. Endometrial ablation cycle is
a. 7-10 days
b. Combined oral
b. 1-3 days
contraceptives
c. 1-2 days
c. Progesterone only pills d. 1-4 days
d. Tranexam e. 2-7 days
e. Mefanemic acid
All are true about primary dysmenorrhea
except:
a. +in both ovulatory & non ovulatory cycles
b. starts at the 1st day of menses,
subsides by the 3rd
c. associated with vomiting & diarrhea
d. caused by prostaglandins increase in the
2nd half of the cycle
Amenorrhea
Which of the following statements about primary
amenorrhea is false:
A- Primary amenorrhea is absence of
menses and secondary sexual
characteristics at the age of 16
All are true about primary amenorrhea except :
B- Gonadal dysgenesis is the most common
A- GONADAL dysgenesis is the most common cause
cause of primary amenorrhea
B- imperforate hymen is the most common
C- In imperforated hymen, blood collects in
C- ovarian ruptured cysts need immediate surgical
vagina
treatment
D- Imperforate hymen can be corrected by
D- Primary amenorrhea is absence of menses and
surgery
secondary sexual characteristics at the age of 16
E- A karyotype is the next step in a patient
with elevated FSH and normal breast
development
Answer: Primary amenorrhea is absence of
menses and secondary sexual characteristics at
the age of 16

All are causes of primary amenorrhea except : One of the following is NOT a cause of delayed
a. ashrmann syndrome puberty:
b.Turner's Syndrome a. Cold climate
c. mullarian agnesis b. Hypothyroidism
d.kallman syndrome c. Prolactinoma
e. gonadal dysgensis Answer: Cold climate
Answer: A

A woman who suffers from anterior pituitary All are true regarding mullerian agenesis except:
failure (Sheehan’s syndrome) can be A. Females affected have short vagina
induced to ovulate by: B. They have normal breasts and pubic hair
a. HMG injection C. They have no uterus
b. Pulsatile GnRH D.t's associated with renal and skeletal abnormalities
c. Bromocriptine E. Need supplemental hormones as part of treatment
d. Clomiphene citrate
Answer: E
e. Low dose estrogen
Answer : A
Regarding premature ovarian failure all of the
All of the following are features of PCOS except:
following is true except:
A. Increased risk of DM type II
A- Autoimmune destruction is a cause
B. Increased acne
B- It is present before 40 years
C. It is a cause of anovulatory amenorrhea
C- It is diagnosed by finding hypogonadotropic
D. Increased levels of 17α-Hydroxyprogesterone
hypoestroginsm
E. They are at increased risk of atherosclerotic diseases
Answer: C
Answer: D
All of the following are true about a patient with Regarding premature ovarian failure, one of the
androgen insensitivity syndrome following is true:
except: a. Occurs in 15% of women
a. The patient will have normal external male b. Should be confirmed by ovarian biopsy
genitalia c. It is unlikely to be caused by infection
b. Patient will have a karyotype with XY d. It is unlikely to occur before puberty
Amenorrhea
c. It is an androgen receptor mutation e. May be caused by previous surgery
d. the patient has no uterus Answer: May be caused by previous surgery
e. The gonad should be removed after puberty
Answer: The patient will have normal external
male genitalia
all of the following regarding secondary amenorrhea
are true except :
All cause secondary amenorrhea except:
a-turner mosaic
Answer : Turner
b-transverse vaginal septum
Answer : ?
Androgen insensitivity true except
false about mullerian agenesis a) small breast
A-no ovary (ovary replaced by fibrous tissue ) b) genotype xy
c) streak of gonad
d) ttt is removal of gonad
e) absent uterus short vagina
Not a cause of secondary
amenorrhea: a. +ovarian dysgenesis
b. anorexia nervosa
Regarding secondary amenorrhea, all are true
except: Regarding Rokitansky syndrome, all are true except:
a. A high serum FSH levels indicate a. The patient is in need for supplement hormones
premature ovarian failure b. Presents with primary amenorrhea
b. Asherman’s syndrome is c. Associated with renal and skeletal
recognized cause abnormalities
c. Pregnancy is the most common d. Is the congenital absence of the vagina and
cause uterus
d. May be caused by Sheehan syndrome e. Presents with normal breast development and
e. It is the absence of menses for more normal axillary and pubic hair
than six cycles interval if previously regular
Answer: E
Which of the following statements about primary
amenorrhea is
Premature menopause may be related to false
all of the following, except: a. Primary amenorrhea is absence of menses and
a. Radiotherapy secondary sexual characteristics at the age
b. Autoimmune of 16
disorder b. Gonadal dysgenesis is the most common
c. Endometriosis cause of primary amenorrhea
d. Mumps c. A karyotype is the next step in a patient with
e. Bilateral salpingoopherectomy elevated FSH and normal breast development
Answer: C Answer: Primary amenorrhea is absence of menses
and secondary sexual characteristics at
the age of 16

Menarche usually occurs


between:
Amenorrhea
a. 14 and 16 years A 28 year old patient complains of amenorrhea after
b. 6 and 8 having dilatation and curettage, the most
years likely diagnosis is
c. 8 and 10 a. Anorexianervosa
years b. Kallman’s syndrome
d. 17 and 18 c. Pelvic inflammatory
years disease
e. 11 and 13 d. Asherman’s syndrome
years e. Turner’s syndrome
Answer: 11 and 13 years Answer: d. Asherman’s syndrome
Not a cause of primary amenorrhea:
a. turner syndrome
b. mullerian agenesis
c. +asherman's
syndrome
d. androgen insensitivity
e. prolactin secreting adenoma
Infertility
Minimal time required before repeating semen
all are IVF commplications except :
analysis for spermatogenesis:
a- OHSS
A- 15 days
b- Ectopic pregnancy
B- 30 days
c- Multiple pregnancy
C- 60 days
d- Gestational DM
D- 90 days
E- ovarian cyst
E- 10 days
answer : D
Answer: D
female with hypogonadotropic hypogonadism All of the following are routine investigations for
seeking fertility which is the best infertility except:
step in her management: A. Progesterone at 21 days in a 28-day cycle
A- LH and FSH B. Semen analysis
B- Clomiphene citrate C. Tests of tubal patency
C- low dose estrogen D. Day 2 FSH, LH, testosterone, FBS
D- bromocrptien E. Day 21 FSH and LH
Answer: A Answer : E
The next step in the management of a patient
with severe
Regarding clomiphene citrate, which is false:
dysmenorrhea and infertility, and normal pelvic
a. can cause hirsutism
U/S is:
b. May cause hot flushes
a. Laparotomy
c. It is given IV
b. HSG
d. Ideally should not be used more than 12 months
c. MRI
e.It can cause ovarian hyperstimulation
d. Laparoscopy
Answer: C
e. CT scan
Answer: Laparoscopy
couple with infertility characterized by semen analysis
Regarding infertility, which of the following is with a sperm count of 14
false: million/ml, 25% motility and 23% normal forms,
a. Ovulation occurs 26 hours after LH peak present to your clinic. The
b. Asthenospermia is the medical term for remaining routine tests are normal. What is the most
abnormal morphology appropriate initial therapy?
c. Seminal fluid makes up to 90% of the a- insemination with donor sperm.
ejaculation b- IVF
d. Endometriosis may interfere with tubal motility c- Intrauterine insemination with washed husband's
e. Cystic fibrosis may lead to congenital bilateral sperm.
absence of vas deferens d- varicocelectomy.
Answer: Asthenospermia is the medical term for e- clomiphene citrate.
abnormal morphology
Answer : A or B
Sperm capacitation refers to the process by which the
spermatozoa become capable
Regarding infertility, all of the following are
of:
correct except:
a. Fertilizing the ovum
A- define as 24 months of failure conception
b. Penetrating the cervical mucus
B - male should abstain 72 hour before semen
c. Stimulating meiosis of the ovum
analysis
d. Producing acrosomal enzymes
Answer: A
e. Dispersing the zona radiate
Answer: Producing acrosomal enzymes
Infertility
Not a recognized indication for All are true about clomiphene citrate except:
IVF: a. causes hyper-stimulation syndrome
a. +septate uterus b. causes hot flushes
b. obstructive azoospermia c. +indicated in premature ovarian
c. severe oligospermia failure
d. unexplained infertility d. increases levels of gonadotropins
e. tubal blockage e. increases possibility of dizygotic pregnancy
All of the following are true about PCOS
except
A young patient who is infertile due to
a. It predisposes to weight gain
hypogonadotropic hypogonadism. What is
b. FSH levels increase
the best treatment for her infertility?
c. Patient can have anovulatory amenorrhea
a. Clomiphene
d. It is associated with Type II diabetes mellitus
citrate
e. It is possibly associated with a risk of
b. LH and FSH
atherosclerosis
Answer: LH and FSH
Answer:

Polycystic ovarian syndrome is characterized by


Regarding IVF, all of the following are indications
all of the following except:
except:
a. Obesity
a. Absent or blocked fallopian
b. Increased
tubes b. Ovarian failure
androgen levels
c. Oligomenorrhea c. Severe male factor
d. Increased LH:FSH infertility d. Severe
ratio endometriosis
e. Decreased e. Infrequent ovulation after failed treatment with
other therapies
estrogen levels
Answer: B
Answer: E
A couple presented with infertility, semen analysis
76. The initial evaluation of an infertile couple shows a count of 14 million, motility
include: a. Semen analysis . 20% and morphology of 24%. The most appropriate
b. Femal action is
karyotyping. a. Varicocolectomy
c. Ovarian b. Insemination with donor
biopsy sperm
d. Hysteroscopy and D&C. c. Clomiphene citrate
e. Antisperm antivodies assay. d. IVF
Answer: A e Intrauterine insemination with washed husband
sperm
All are considered normal in semen analysis,
except:
a. morphology 50% normal
b. +motility 20% forward motile
c. liquefaction within 20 minutes
d. volume 4 ml
e. density 20 million/ml
Infertility
Contraception
Progesterone releasing IUCD all are true except:
all absolute contraindication of IUCD except :
A- inhibit fertilization as a part of it is mechanism
a- Active PID
B -contraindicated in breast cancer
b- unexplained vaginal bleeding
C- can be inserted for 5 years
c- History of ectopic pregnancy
D- can treat functional cyst
d- trophoblastic malignant tumor
E- It's labled by the FDA as treatment for
e- Uterine anomaly.
mennohragia
A : c ( not sure )
Answer: D
Absolute contraindication for OCP include all of the progesterone-only contraception is a present in all
following except: of the following except :
A. Diabetic retinopathy a-oral
B. Non focal migraine b-implant
C. Active liver disease c-transdermal patch
D. Complicated valvular heart disease d-IUCD
E. Smoker , with age more than 40 e-IM injection
Answer: B A:C
Combined oral contraceptives are contraindicated in
all of the following except
a. Migraine headache with focal headache COCP all true except:
b. Severe liver cirrhosis Answer : decrease risk for cervical cancer
c. IHD
d. Diabetes mellitus
Answer: D
Progesterone-secreting IUD is used in all of the
following except:
About COCPs, one is correct:
a. intramural
a. should be taken with periodic pill breaks
fibroid
b. +doesn't affect future fertility
b.
c. cause acne
endometriosis
c. DUB
Combined oral contraceptives decrease the risk
of all of the following except
Which of the following reduces the efficacy of a. Benign breast
COCPs: disease
a. b. Salpingitis
+rifampicin
c. Colorectal
b. ranitidine
cancer
c.metochloprami
d. Endometrial
de d. steroids
cancer
e. hydrochlorothiazide
e. Cervical cancer
Answer: Cervical cancer
The most likely cause of acute abdominal All of the following are absolute contraindications
pain immediately after an IUCD insertion for combined oral contraceptive
is: pills, except;
Contraception
a. Uterine a. Breast cancer
Perforation b. Previous pregnancy while on
b. Uterine pills c. Previous DVT
contraction d.Thrombophi
c. Spontaneous expulsion of the lia
device e. Liver
d. Copper allergy adenoma
e. Pelvic inflammatory disease Answer; B
Answer: A
All of the following are progesterone only
Combined oral contraceptives are contraindicated
contraception except:
in all of the following except
a. Norplant.
a. Migraine headache with focal
b. Implanol.
headache
c. Mirena coil.
b. Severe liver cirrhosis
d. Lippes loop
c. IHD
e. Depo-
d. Diabetes mellitus
provera.
Answer: D
All of the following are side effects of
progesterone only contraceptives except
a. Functional
cysts
b. Acne
c. Weight gain
Menopause
in postmenopausal woman all investigations can be
done except : In postmenupausal bleeding, one is
a- transvaginal ultrasound not mandatory:
b- estrogen a-Speculum exam
c- cervical inspection b-Vulva inspection
d- Pap smear c-US
e- valvular examination A:
A:b
Regarding postmenopausal bleeding,
which is false:
9. Hormonal replacement therapy increases the risk of a. Endometrial thickness of 0.5cm is
all of the following except: an indication for biopsy
a. Stroke b. Endometrial CA is the most
b. Alzhemier’s disease common cause
c. Thromboembolic disease c. May be due to vaginal atrophy
d. Breast cancer d. Cervical CA is a recognized cause
e. Endometrial cancer e. The incidence is higher at age 80
Answer: compared to age 50
Answer: Endometrial CA is the most
common cause
Hormonal replacement therapy can
The age of menopause is: treat all of the following except:
a. Genetically determined a. +coronary artery
b. Affected by OCP use
disease b.
c. Smoking has no effect
osteoporosis c.
d. Affected by the age of last pregnancy
vaginal dryness d. hot
Answer: Genetically determined
flushes
e. mood swings
All of the following are symptoms of
The median age of menopause is a. 47 menopause except
b. 51 c. a. Hot flashes
45 d. b. Loss of weight
41 e. c. Insomnia
57 d. Anxiety
Answer: b. 51 e. Genital prolapse
Answer: Loss of weight
Symptoms of menopause include all of the Regarding cervical ectropion, all of
following except: the following are true except :
a. Infertility a. It is commonly seen in
b. Stress incontinence post-menopausal women.
c. Loss of libido b. Cervical smear is
d. Breast enlargement mandatory before treatment.
e. Dry vagina c. Post coital bleeding is a
Answer: D common presentation.
Menopause
d. Can be caused by
combines contraceptive pills.
e. Excessive mucoid secretion is a
common presentation.
Answer: D
HRT is beneficial in the management of patients with
all of the Regarding menopause which
following except: of the following statements is
a. Mood change incorrect
b. Vaginal atrophy a. Average age is 55
c. Coronary heart disease b. Delayed menopause is a risk
d. Flushing factor for endometrial cancer
e. Osteoporosis c. Smoking induces early menopause
Answer: C
About post menopausal bleeding, all
are true except:
All are risk factors for premature ovarian failure a. endometrial thickness >5 mm on
except: ultrasound is an indication for biopsy
a. mumps b. vaginal atrophy is a recognized
b. radiation cause
c. autoimmune disease c. +endometrial cancer is
d. +progesterone mini pills the most common cause
e. smoking d. cervical carcinoma
should be considered

All of the following are risk factors for osteoporosis


except:
a. +late menopause
b. low BMI
c. smoking
d. GnRH use
Obstetric & Gyne Procedures & Pap Smear
all can be seen on Hystrosalpingogram except:
Regarding hysteroscopy, all the following
a- Endometrial polyp
are true except:
b- Uterine adhedions
A-You can see the ostia of the tube
c- Subserosal fibroid
B-Best treatment for intramural fibroid
d-patency of Fallopian tube
Answer: B
Answer : C
All of the following can be seen through Which one of the following is true in regard to a
a HSG except: cervical Smear with ASC-US:
A-Asherman's A- Best combined with HPV triage
B-Subserosal fibroid B- Refer to colposcopy
C-Septate uterus C- Treated by conization
D-Proximal tubal obstruction (ostium) D- Should be treated as CIN I CIN II
E- assess the contour and the adequacy E- It's normal finding and nothing should be
of the uterine cavity. done (something like this)
Answer: B Answer: A
Others
All are contraindications of vaginal hysterectomy
all of the following causes fetal
except:
tachycardia except :
A- Cervical cancer
a- Maternal fever
B- Large cystocele
b- Pethidine
C- Tight vagina
c- Anemia
D- Previous 3 CSs
d- fetal distress
E- Ovarian cancer
Answer : b
Answer: D
All of the following regarding
gynecological surgeries are true except:
A. Uterosacral ligament could be viewed Regarding cervical incompetence, all the following
in laparoscopy are true except
B. Fallopian Ostia could be viewed by a. Can be caused by multiple pregnancy
hysteroscopy b. Can be diagnosed by ultrasound
C. Preprocedural counseling is optional c. Is a cause of preterm labour
D. Total hysterectomy includes the d. Can be caused by cone biopsy of the cervix
removal of uterus and cervix only e. Best treated in the 2nd trimester of pregnancy
E. Subserosal fibroids could be removed Answer: A or E
by laparoscopy
Answer: C
Congestive (secondary) dysmenorrhea can One of the following is not used in the management
be caused by all of the following of acute PID:
except a. Clindamycin
a. Regular use of COCP b. Laparoscopy
b. Cervical stenosis c. Oral doxycycline
c. Pelvic inflammatory disease d. D&C
d. Adenomyosis e. Removal of IUCD
e. Asherman’s syndrome
Answer: D&C
Answer: Regular use of COCP
One of the following is not associated
with polyhydramnios: all can be caused by Sheehan’s syndrome except:
a. Twin-Twin transfusion syndrome a-hypoglycemia
b. Diabetes b-amenorrhea
c. Esophageal atresia c-lactation
d. Vasa Previa d-fatigue
e. Anencephaly answer : c
Answer: Vasa Previa
Age of menarche : Mrs. A presented to the clinic complaining of
a-14 16 painless vaginal bleeding after 8 weeks
b- 8 10 of amenorrhea. She delivered vaginally 6 months
c-11 13 ago and currently she is lactating. B-
e-16 18 HCG found to be (1200 IU/L). Transvaginal U/S
Answer : C showed an empty uterus. One of the
Others
following is correct regarding her case:
a. Diagnostic laparoscopy should be the first step in
her management
b. Repeating B-HCG after 7 days is advised
c. This picture might represent an early intrauterine
pregnancy
d. Complete molar pregnancy is a strong possibility
e. Methotrexate injection is the ideal way of
managing her
Answer: Both B and C seem to be correct!
About pelvic surgeries, which of the following is
mismatched
One of the following doesn't cause
a. Posterior colporrhapy – Rectocele
hirsutism:
repair b. Cone biopsy – vaginal
a. Cyproterone citrate
procedure

The following are most likely causes for abdominal


pain at 20 weeks gestation except
One of the following is not a vaginal a. Ectopic pregnancy
procedure: b. Spontaneous
a. anterior colporrhaphy miscarriage
b. +ovarian cystectomy c. Complicated ovarian
c. resection of Bartholin cyst cyst
d. vaginal hysterectomy d. Acute appendicitis
e. Urinary tract infection
Answer: a. Ectopic pregnancy
Which of the following procedures
assesses both uterine cavity and
tubal lumen:
A- HSG .
B- Laparoscopy.
C- Hysteroscopy .
D- Ultrasound.
E- Colposcopy.
Answer: A

1-

1. Anemia , thromboembolism and bleeding disorders


1-Which of the following is NOT true?

- Warfarin should not be used during breastfeeding

2- Which of the following is true ?

A. warfarin can cross the placenta


B. warfarin is secreted in milk
C. heparin can cross the placenta
D. heparin is secreted in milk
E. aspirin is a strong anticoagulant
3- Most common anemia in pregnancy:

A. folate deficiency
B. IDA
C. sickle cell anemia

4- Regarding Antiphospholipid syndrome .. which of the following statements is incorrect?

A. Associated with lupus anticoagulant, ds-DNA, ANA antibodies


B. Arterial and venous thrombus can occur
C. Can be acquired
D. Continue anticoagulants for 6wks postpartum
E. Cause early onset of preeclampsia

5-Antiphospholipid syndrome has a recognized association with all of the following except:

A. Early onset preeclampsia


B. IUGR
C. IUFD
D. Placental Abruption
E. Macrocosmic baby
6-

7-Regarding IDA , which of the following is False ?

A. RBCs are Macrocytic hypochromic


B. RBCs are microcytic and hypochromic

8- Regarding the treatment of anemia in pregnancy, all of the following are true, except:
A. 60 mg of iron is adequate dose in pregnancy
B. Parenteral iron is used to correct anemia faster
C. 0.4 mg folate is required for all pregnant ladies as a minimum
D. Gastrointestinal side effects are the main cause of noncompliance to therapy
E. With optimal treatment, average Hb rise is 0.8 g/dl per week

9-Which of the following is incorrect?


A. in mechanical valve continue LMWH
B. LMWH causes osteoporosis
C. Aspirin is not anticoagulant
D. Heparin & warfarin not secreted in milk

10- Question about DIC All except :


A. placental Abruptionn can lead to it
B. platelets increase
C. prothrombin time decrease

2. Other medical disorders


1-13 yrs old girl with primary dysmenorrhea , best management ?

- anti prostaglandin ( ponstan )

2- what is false about the Mayer-Rokitansky-Kuster-Hauser syndrome.

A. ovaries replace with fibrous tissue


B. absent uterus short vagina
C. 46xx karyotype
D. normal pubic hair and breast development
3- Regarding Thyrotoxicosis all of the following are true except:
A. prophylthiouracil cause Goiter
B. Clinical is unreliable
C. High thyroxine is diagnostic
D. Cause preterm labour

3. Drugs in pregnancy , tocolytics , analgesia and anesthesia

1-Spinal anesthesia vs epidural what is false ?

A. More headache
B. More hypotention
C. Needs larger dose
2- Question about steroid

A. Inhaled steroid is highly teratogenic


B. Systemic steroid can cause orofacial deformity if used in 1st trimester
C. Statement about dexamethasone injection..
D. Statement about topical steroid
3-Which of following is Contraindication of epidural anesthesia ?

- Answer : platelets < 50,000

4-G2P1 presented with 5 cm cervical dilation at 36 weeks and regular contraction every 10 mins ?

A. expectant management
B. tocolysis only
C. tocolysis+corticosteroid+antibiotics
D. corticosteroids and antibiotics
E. tocolysis and corticosteroids

8- regarding opioids , which is NOT true ?

A. can cross placenta


B. fentanyl is the most common used
C. continuous infusion is preferred
D. naloxone is antagonist
7- correct about epidural anesthesia
A. cause headache
B. contraindicated in DVT

8- Which of the following is false :


Answer: tocolytic for all pt

9- A drug that has had adverse effects on animal testing and no proved benefits....which category is it?
A. C
B. A
C. B
D. X
E. D

10- Contraindication of epidural anesthesia ??


A. Primigravida
B. platelet below 60,000

4. PPH

3-Which is NOT to be given during postpartum hemorrhage ?

- Answer: PGE2 ( because it causes vasodilation )

4- Which of the following is NOT a risk factor for PPH ?

A. meconium
B. Instrumental delivery
C. Multiparty
D. APH
E. anemia
5- Regarding Primary PPH, which is true:

A. Active management of the 3rd stage of labor will reduce the incidence
B. Defined as vaginal bleeding that occurs one week after vaginal delivery
C. More common in vaginal delivery than C/S
D. Endometriosis is the commonest cause
E. Defined as vaginal bleeding of 200mL past vaginal delivery
6-2 weeks of continued bleeding after normal not complicated vaginal delivery ,most common cause :
A. RPOC
B. coaugulopathy
C. atony
D. laceration

7- When does the uterus become a pelvic organ after delivery?

A. 2 weeks
B. 1 week
C. 2 months
D. 1 months
E. 6 weeks
8- PPH after 2 weeks :
Answer : Retained products of conceptions

5. Peurperium

6. Antepartum hemorrhage and obstetrics emergency


1-Incidentally discovered major placenta previa , Which of the following is NOT steps of management ?

A. Vital signs
B. Correct anemia
C. Iv cannula
D. Steroid
E. Delivery

2- Which of the following is correct regarding APH :

A. Vasa previa is rupture of fetal blood vessels


B. placenta previa diagnosed early as 22 week
3-Regarding placenta previa , which of the following is false?
Answer: dx by clinical exam
5- Regarding APH which of the following is incorrect?
A. most common is placenta previa
B. cervical polyps cause APH
C. steroids are given to preterm babies
D. ultrasonic are done for all patient with APH

7. Fetal malpresentation

1-Regarding malpresentation, all of the following are true except :

- Persistent brow presentation is an indication to use forceps.

2-question about flexed vertix :

- sub-occopito-bregmatic

3- ALL are cause of face presentation except:

A. AAnencephaly
B. Branchial cyst
C. Multiple pregnancy
D. Multiparty
E. contracted pelvis
4- Regarding malpresentation , Which of the following is Not true ?

- Mode of delivery in brow presentation is by forceps vaginal delivery

5- Regarding fetal denominator, which is false:

A. Occipito-Frontal diameter ---> Vertex presentation


B. SubMento-Bregmatic diameter ---> Hyperextended
C. Mento-Vertical diameter ---> Brow presentation
D. SubOccipito-Bregmatic diameter ---> Vertex presentation
E. Biparietal diameter ---> 13cm
6- Regarding malpresentation, all of the following are true except :
A. forceps can be used in breach
B. Forceps can be used in all presentations and is indicated in prolonged stage 2

7- How to hear breech fetal heart ?


A. above umbilicus opposite back
B. below umbilicus opposite back
C. below umbilicus at side of back
D. Above umbilicus at side of back ( NOT sure )
E. Eat umbilicus
8- All of the followin are risk factors for breech presentation except
A-oligohydroamnios or polyhydroamnios ( both in same choice)

9- One is false
A. android pelvis >>> males
B. anthropoid pelvis >> obstructed labor
C. Android pelvis - persistent O.P position?
D. Gyencoid---- ovoid and transever

10- Mismatch :
A. transverse - lie
B. face – presentation
C. breech – presentation
D. vertix - position
E. altitude – flexion

8. Normal and abnormal labor


1- Which of the following is NOT in normal labor management :

A. Check vital signs


B. Hydration
C. check Blood glucose

2- About labor , what's wrong ?

A. latent phase is a part of secondary stage of labor


B. 6 cm dilation is first stage
3- Regarding labor , which of the following is true ?

A. Latent phase is a part of secondary stage of labor


B. 6 cm dilation is first stage

4- Regarding preterm labor, which is false?

A. Antenatal corticosteroids are associated with increased risk of maternal hyperglycemia


B. Ritodrine (as a tocolytic) has higher maternal side effects in relation to nifedipine
C. Indomethacin is associated with increased risk of premature closure of ductus arteriosus
D. Indomethacin may be the first line tocolytic in cases associated with oligohydramnios
E. Long term maintenance doses of tocolytics has no proven effect
5- Which of the following is not a risk factor for preterm labor:

A. Smoking
B. Oligohydrominos
C. Bacterial vaginosis
D. Age more than 35 year
E. Anemia
6- Regarding 3rd stage , which of the following is false?
A. prologed third stage mean more than 1 hour
B. Active management is done if third stage more than 1 hour
Note : Other form the answer was
The head is engaged if the station above 0

7- Which of the following is false about stages of labor ?


A. ask pt to push as soon as cervix is fully dilated
B. Primi with epidural less than 3 hours

8- Question about engaging diameter:


A. occiptofrontal>>10
B. Verticomental>>13.5
C. Other choices can’t remember
D. suboccipito-vertical - 9.5
E. biparietal – 10

9. Delivery ( vaginal , assisted & CS )

1-Twin delivery which is false ?

A. it the second baby is in transverse lie emergency CS should be preformed


B. Episetomy isn’t routinely used
2-Safe vaginal delivery in ?

- its safe in anterior coloporaphy repair

3-all are NVD except ?

- GA 36wks + 6 days

4-Not an absolute indication of CS ?

- Answer : cord prolapse with a dead fetus

5- Not contraindication of CS:

A. Repaired prolapse [Anterior colopraphy]


B. 2CS
C. One classical CS
D. Myomectomy
6- About shoulder dystocia, NOT true:

A. Episiotomy to release posterior shoulder


B. DM is a risk factor
C. Robin to decrease shoulder to shoulder diameter
7- Which of the following is true about CS ?
Answer : CS is safe when platelet 70000

8- About twins all of the following is indication for CS Except?


Answer : 1st cephalic 2nd breech

10. ANC

1-Which of the following heart conditions carries the worst prognosis during pregnancy:

A. Mitral stenosis
B. Aortic stenosis
C. Coarctation of the aorta
D. Eisenmenger syndrome (Ans)
E. VSD

3- Drug contraindicated in pregnancy

- Nitrofurantoin

4- Regarding cholestasis , which of the following is NOT true ?

- Causes specific skin lesions

5- Not a cause of symmetrical IUGR:

A. Placental insufficiency
B. TORCH
C. Congenital anomalies
D. Constitutional
6

7-All of following are cause for Small GA except:

A. long standing DM
B. smoking
C. oligohydramnios
D. dietary pregnancy
8- pregnant after previous delivery and she was lactating , what is the best method to measure her EDD :

A. US in 1st trimester
B. US in 2nd trimester
C. LMP
9- LMP 29/12/2017 .. EDD is :

A. 5/10/2018
B. 22/10/2018
C. 1/9/2018
D. cant be calculate
10- Which of the following is NOT a cause for large GA in first trimester:

A. wrong date
anti phospholipid
B. polyhydramnios
C. full bladder

11- All can be diagnosed by us in 11 week gestation except:

A. Trisomy 21
B. Fetal position
12- Concerning small for gestational age fetus, which is wrong:

A. Could be constitutional
B. Has an association with fetal infection
C. Once diagnosed, it is an indication for delivery
D. Approximately, half of the cases are identified on routine abdominal palpation
E. Has an association with fetal abnormalities
13-A woman come for her first antenatal visit, previous history of Singleton pregnancy at 39W, another at 28W
and one ectopic pregnancy. What is the GP?
- Answer : G4P2+1

14- what causes IUGR ?


Answer : long standing DM

13- All of the following protect fetus from the drug except:
A. Highly ionized
B. Highly protein bound
C. Low maternal blood concentration
D. Highly lipophilic
E. Larger than 1kD

14-7 week all visible in US except


A. Empty uterus
B. Two gastrointestinal sac
C. Fetal echo
D. Fetal heart activity
E. Corpus luteal cyst
I found it 2 Ans!!

15- All of the following are true acc to ANC except :


A. Seronegative >>> rubella vaccine after delivery
B. ANC decrease post term pregnancies
C. Pelvic exam done routinely
D. Urine dipstick for protein every visit
16- Patient with growth restruction:
A-Most common presentation Polyhydroamnios

17- EDD 24/3/2019:


A. 31/12/2019
B. 31/1/2020

11. Fetal abnormalities, growth and monitoring

1-Regarding CTG , Which of the following is NOT true ?

A. Variability <5
B. Bpm 100-160
2-Which of the following has NO increase in recurrence ?

- cord prolapse

3- Regarding malpresentation , Which of the following is Not true ?

- Mode of delivery in brow presentation is by forceps vaginal delivery

4-pregnant in her 20 GA , there is no fetal heart activity best method of treatment :

A. Methotrexate and misoprostol


B. misoprostol alone
C. Dilatation and curettage
5- Which of the following is not one of the normal parameters of CTG?

A. Variability >10
B. early decelerations
C. Baseline 140 bpm
D. 2 accelerations in 10 minutes
E. 3 accelerations in 20 m
6- All in biophysical profile except:

A. Fetal tone
B. Fetal breathing
C. CTG
D. Liquor volume
E. Doppler
7- cord entanglement most commonly seen in which type of twins

A. Monoamniotic monochorionic
B. diamniotic monochorionic
C. diamniotic dichorionic
D. conjoined twin
8- Which of the following is NOT a component of biophysical profile:

A. CTG
B. Amniotic fluid volume
C. Fetal tone
D. doppler US
E. fetal movement
9- All of the following are true about CTG expect ?
A. early deceleration —> opioid
B. late deceleration —> hypoxia
C. variable deceleration —> cord compression

10- All of the following are indication of CTG except:


A. In oxytocin induction
B. In previous CS
C. APH
D. Hx of PPH

11-

12- All of the following need continuous CTG except ?


A. Previous PPH
B. Previous CS
C. Fetal distress

12. Urinary incontinence & genital prolapse

1-About uterine inversion one is false :

- chronic inversion occurs 1 week after delivery (it occurs one month after delivery)

2-One is false regarding uterine inversion:

A. Traction on the peritoneal structures lead to vagal response


B. Non_purpural uterine inversion can be caused by uterine tumor
C. Chronic inversion happens after 1 week of delivery (Ans) (4 weeks not 1 week)
D. It is caused by traction on the placenta
E. JOHNSON'S maneuver is used in the treatment of uterine inversion
3

5- Regarding stress incontinence , one is false:

A. sphincter damage is the most common cause


B. electrical smth is one line of treatment
C. subvesicural smth is the most common procedure is done for treatment
D. the most common incontinence
E. involuntary leakage due to coughing sneezing
6-Which of the following is NOT a surgery for genital prolapse ?

- Answer: abdominal hysterectomy

7- All of the following can be used in the management of urge incontinence except:

A. bladder training
B. avoiding caffeine drinks
C. sacral nerve root neuromodulation
D. tolterodine and oxybutynin
E. Anterior vaginal colporrhaphy

9- All of the following is correct about uterine inversion expect ?


Answer : management by laproscope /surgery

10-
13- False about urge incontinence :-
Answer : Management by Retropubic bladder neck suspension operations

13. Fibroid

14. Endometriosis, Adenomyosis & Endometrial CA

1- What is The definite diagnosis of endometriosis :

A. Hysteroscopy
B. Laparoscopy

2- Regarding endometrial cancer, which of the following is false?

A. Pelvic pain is a late symptom is the disease process


B. Well differentiated Endometroid type tends to have good prognosis
C. The incidence in increasing worldwide
D. Clear cell carcinoma is typically diagnosed premenopausal
3- Endometroid one is true :
A. Defective Diagnosis by histopathogy
B. severity related to extend of the disease
4- Which of the following is Not risk factor of endometrial CA ?
Answer : COCP

5- all germ cell tumor except :


Answer : endometrioid

6- all poor px factor except :


Answer : endometrioid

7- a 45 years old female with bleeding what is expected to be found in her uterus:
A. fibroid may be found in postmenopausal
B. endometrial thickness more than 4 mm is alarming
8- Regarding endometrial hyperplasia, what is the most indicate malignancy ( or something like that):
A. cellular atypia
B. mitotic rate
C. age
D. Race
9- One is correct about endometriosis :
A. commenly presented with menorrhagia
B. definitive diagnosis by histopathology
C. symptoms correlate with disease severity

15. Cervical dis & CA

1- Regarding cervical ca , Which of the following is true?

- Screening should be started at 21 year regardless onset of sexual activity

2- Which of following is Not true about cervical ca:

A. Stage 1b1 treated by chemoradiation


B. 30% AdenoCa
3-Which is NOT true about cervical ectropion ?

- Answer: caused by mini pills

4- Bishop score measures all of these parameters except:

A. cervix consistency
B. cervix position
C. cervix dilatation
D. station of the head
E. position of the head
5- All of the following are protective factors against cervical cancer except :
A. HPV vaccine
B. Condom
C. quitting smoking
D. COCP
6- Regarding cervical CA , Which of the following is NOT true?

- Answer: Simple hysterectomy involves removal of uterus cervix and parametrium ( Simple hysterectomy doesn't
involve the parametrium)

7- Regarding Cervical CA, what is false:

A. It can prevented by the use of condom


B. HPV 16 & 18 are found in over 70% of all cervical cancers
C. History of vulvar or vaginal squamous intraepithelial neoplasia is a risk factor
D. The natural course of the disease is well known
E. The ovaries should be removed during surgery
8- Which of the following is incorrect about cervical Cancer 1b1 ?
Answer : treated mainly by chemotherapy and radiotherapy

9- Regarding Cervical cancer which one is wrong ?


A. cone biopsy for early stage
B. tha cholectomy is indicated when patient wants to preserve fertility and tumor < 6 cm
C. after surgery, if recurrence give chemotherapy
D. radiation and chemotherapy lead to more sexual dysfunction than surgery

10- Cause of cervical cancer :


Answer : HPV

16. Ovarian CA
1-ovarian cancer associated with BRCA1

- High grade serous ovarian cancer

2-Elevated LDH with decrease in HCG and AFP.

- Dysgerminoma.

3-Elevated Inhibin is seen in which Ca?

A. Mucinous
B. High grade serous
C. Choriocarcinoma
D. granulosa cell tumo
4- Which of the following is Ovarian ca associated with endometriosis:

- Clear cell
5-Which of the following is associated with BRCA1 gene ?

- Serous adeno Ca ( NOT sure)

6-AFP tumor marker of :

- Answer: Endodermal sinus tumor

8-Regarding ovarian epithelial tumors , which of the following is NOT true?

Answers: Mucinous is the most common type

9-Least risk to develop ovarian cancer :


A-Multi-parity , late menopause and take COCP

B-nulliparity and regular period

In other form Answer : +Multiparity with new menopause

17. Vaginal discharge

1-Regarding bacterial vaginosis one is false :

A. no need to treat the partner


B. metronidazole is the DOC
C. Whiff test +ve
D. most common cause is lactobacilli

2-16 year old female started to have sexual intercourse recently. She complained of irritating green frothy
vaginal discharge. The discharge had a pH of 6. What's the most likely organism causing this:

A. Candida
B. Tricomonas vaginalis
C. Syphilis
D. Gardenella E- Chlymida.

3-false about bacterial vaginosis?

- Its sexually transmitted

4-The MO cause bacterial vaginitis :

- Gardnerella vaginalis

5-Which of the following is False about vaginal candidiasis ?

- Answer: usually asymptotic, it’s usually itchy with white discharge


6-Which of the following is NOT true about bacterial vaginosis ?

- Answer: NO vulvar itch

7- Regarding chlamydia trachomatis infection, all of the following are true except:
A. Is a common cause of vaginal discharge
B. May predispose to perihepatic adhesions
C. May be treated with erythromycin or tetracycline
D. Is a gram negative intracellular organism
E. Can cause neonatal pneumonia

Note : Other form the answer is


can be cultured easily from vaginal discharge

8- Which of the following is correct about candida ?


A. mucopurulent cervicitis
B. you can dx by mix discharge with koh

9- True about bacterial vaginosis :


Answer : Fishy odor

18. Menorrhagia

1-regarding DUB , which is NOT true ?

A. Common after menarche


B. copper IUCD use in treatment of DUB

2- DUB, one is false

A. NSADs decrease about 25-50%


B. mini-pills is used in treatment of anovulatory dub
C. tranexamic acid is c/i in bleeding tendency
2-A 33 year female p4+0 , presented with menorrhagia for the first time , All are important investigative for her
Except :

A. pap smear
B. CBC
C. TFT
D. vaginal US
E. endometrial biopsy
3- Which of the following is NOT use in management of DUB?

A. Mirena
B. COCP
C. progesterone mini pills
D. Mefanamic acid
E. Tranexamic acid
4- Regarding asherrman syndrome , which of the following is NOT true ?
A. cause primary amenorrhea
B. Diagnosed by hystroscopy and HSG
C. As a result of D&C
D. Previous CS is a risk factor
E. Management by adhesiolysis

19. Amenorrhea

1-Causes of secondary amenorrhea except ?

A. PCOS
B. Premature ovarian failure
C. Chemotherapy
D. Ovarian cystectomy ( NOT sure )
2-All are true about primary amenorrhea except :

A. GONADAL dysgenesis is the most common cause


B. imperforate hymen is the most common
C. ovarian ruptured cysts need immediate surgical treatment
D. Primary amenorrhea is absence of menses and secondary sexual characteristics at the age of 14
3- All of the following are causes of secondary amenorrhea except:

A. PCOS
B. Hyperprolactinoma
C. androgen insensitivity syndrome

4- All of the following is indication to give clomiphene citrate except ?


Answer : premature ovarian failure

5- Regarding PCOS which of the following is false ?


A. seen in 20 percent of premenopausal woman
B. cause premature menopause
C. Commonly have hyperprolactinemia
D. Increase insulin sensitivity
E. Negative progesterone challenge test
6- All cause secondary amenorrhea except:
A. premature ovarian failure
B. Kallman syndrome

20. Infertility

1-regarding semen fluid analysis , which of the following is false ?

- Motility 25%

2-Wrong about primary infertility ?


- Answer: female factor accounts for 85% of cases

3- Question about semen analysis which is not true according to WHO values:
A. 2 ml
B. 25 million sperm count
C. Morphology 94%
D. Progressive motility 15%
E. Total motility 50%

4- Which of the following is not important to tell the patient in IVF ?


A. Cause of infertility
B. Number of oocyte
C. Number of returned embryos
D. LMP

21. Contraception

1-Regarding COCP , Which of the following is false ?

- It causes acne

2-Most common organism colonizing Copper IUD:

A. Actinomyces ( NOT sure )


B. Staph A
3-False about minipills

- contraindicated in woman > 35 , smoker

4- All are true about COCP except:

A. Reduce risk of endometrial cancer


B. Reduce risk of ovarian cancer
C. Reduce risk of cervical cancer
D. Regulate the period
5- All are absolute contradiction COCP except :

A. Recent thromboembolism
B. Liver cirrhosis
C. Breast cancer
D. Smoking
6-According to copper IUCD all are true except ?

A. 0.5% pearl index.


B. Can remove any time during menstrual cycle
C. contraindication PID
7- About injectable progesterone, one is false
A. High effective way, with failure rate 0.3%
B. increase the risk of VTE
C. effective for 9 month
D. Use in women with sickle cell anemia

8- All true except :


Answer : Diabetic pt can't take MINIPILLS

9- True about COCP except :


Answer : Increase acne

22. Menopause

1- Regarding post menopause bleeding , which of the following is NOT true ?

- the most common cause is endometrial ca

2- Regarding menopause , which of the following is NOT true?

- Smoking cause late menopause

3-One of the following is Not a symptom of menopause :

- Answer : vaginal discharge

4-Most common cause of post menopausal bleeding ?

- Answer: atrophic causes

5- Which of the following is NOT a Risk factors for premature menopause ?

A. Low BMI
B. Multi-parity
C. Smoking
Family history E- Ovarian surgery

6- Which of the following is incorrect?


A. fibroid lead to early menopause
B. Dm1 cause early menopause
C. hot flushes is common

23. Obstetric & Gyne Procedures & Pap Smear

1-Regarding Management of bicoruate uterus , which of the following is true ?


- hyesteroscopic metroplasty ( NOT sure )

2-cervical smears during pregnancy which of the following is false?

- punch biopsy option

3 - Not complication of curretage :

Answer : Ectopic pregnancy

24. Others

1-Question about STD :

- H.ducreyi —> chancroid

2-genital Warts associated with ?

- HPV 6

3-All of the following are STD except :

- Answer: toxoplasmosis

4- Which of the following is NOT complications of PID ?

A. Infertility
B. Dysmenorrhea
C. Dyspareunia
D. ectopic pregnancy
E. cervical stenosis
5-Regarding STDs, which is false:

A. Chlamydia is associated with pelvic adhesions


B. Active genital herpes in labor can deliver vaginally
C. In HIV infection, lactation should be avoided
D. Penicillin is the drug of choice in all stages of syphilis
E. Presence of genital warts requires cervical smear
6- All of the following are STD except :
A. clamedya
B. candida
C. HIV
D. trachomnals
E. gonnheria

You might also like